You are on page 1of 88

7/31/2017 Cardiology - FRCEM Success

Dashboard Subscription expires in: 82 Days Extend

Question Navigator
Cardiology
Question 1 of 24 1 Current Question

A 68 year old man is brought to hospital complaining of severe sharp chest pain which is 2 Unanswered
radiating between his shoulder blades. An ECG has been performed which is normal. A chest x-
3 Unanswered
ray has been performed which is shown below:
4 Unanswered

5 Unanswered

6 Unanswered

7 Unanswered

8 Unanswered

9 Unanswered

10 Unanswered

11 Unanswered

12 Unanswered

Courtesy of J. Heuser JHeuser (own patient) [GFDL


(http://www.gnu.org/copyleft/fdl.html) or CC-BY-SA-3.0
(http://creativecommons.org/licenses/by-sa/3.0/)],
http://intermediate.frcemsuccess.com/rev/cardiology/
via Wikimedia 1/4
7/31/2017 Cardiology - FRCEM Success
(http://creativecommons.org/licenses/by-sa/3.0/)], via Wikimedia
Commons

a. Give two abnormal ndings seen on the chest x-ray and give the most likely diagnosis. (1
mark)
b. Brie y outline the pathophysiology of this condition. (1 mark)
c. You note the patient has unusually long arms compared to his trunk, and has long
‘spidery’ ngers. What additional condition may this patient have which predisposes to
developing this diagnosis? (1 mark)

You did not answer this question

Answer
a. Widened mediastinum and large aortic knuckle – aortic dissection
b. A tear in the tunica intima allows blood to enter the vessel wall, separating (‘dissecting’) the aortic wall
layers with formation of a true and a false lumen.
c. Marfan’s syndrome

Notes

Aortic dissection is de ned as disruption of the medial layer of the aortic wall provoked by intramural bleeding,
resulting in separation of the aortic wall layers and subsequent formation of a true lumen and a false lumen, with or
without communication.

Risk factors:

Hypertension
Pre-existing aortic or aortic valve disease
Family history of aortic disease
Connective tissue disease (e.g. Marfan’s syndrome, Ehlers-Danlos syndrome)
Smoking
http://intermediate.frcemsuccess.com/rev/cardiology/ 2/4
7/31/2017 Cardiology - FRCEM Success
Smoking
History of cardiac surgery
Direct blunt chest trauma

Clinical features:

Sudden tearing/ripping/sharp chest pain, maximal at onset, radiating to the back and interscapular area
Syncope
Hypertension (although hypotension may be present in tamponade)
Asymmetrical blood pressures in each arms
Inequality in pulses on each side
New diastolic murmur (aortic regurgitation)
Pericardial friction rub
Neurological de cit

Dissection may be classi ed as Stanford type A or B:

Type A involves the ascending aorta and/or aortic arch (35% mortality) – usually treated surgically
Type B involves only the descending aorta and arises distal to the origin of the left subclavian artery (15%
mortality) – usually treated medically

Investigations:

CXR
Widened/abnormal mediastinum
Double knuckle aorta
Left pleural effusion
Deviation of the trachea to the right
Separation of two parts of the wall of a calci ed aorta by > 5 mm
ECG – may show ischaemia as a consequence of aortic dissection
Echo
CT aorta – de nitive diagnosis

Complications:

Acute coronary ischaemia


http://intermediate.frcemsuccess.com/rev/cardiology/ 3/4
7/31/2017 Cardiology - FRCEM Success

Neurological de cit
Limb ischaemia
Cardiac tamponade
Aortic regurgitation
Haemothorax
Stroke
Aortic-oesophageal stula
Retroperitoneal haemorrhage
Mesenteric ischaemia
Renal failure

Next > Submit Something wrong?

FRCEM Success Resources


We are an online revision resource for The Royal College of Emergency Advanced Life Support Group
FRCEM Primary and Intermediate Medicine Emergency Medicine Journal
exam preparation. Irish Association for Emergency Lifeinthefastlane
Medicine Instant Anatomy
Terms & Conditions Advanced Trauma Life Support Patient.co.uk
Get in Touch Resuscitation Council (UK)
TeachMeAnatomy
Trauma.org
Radiopaedia

©2014 - 2017 FRCEM Success | Website designed & hosted by Cyberfrog Design

http://intermediate.frcemsuccess.com/rev/cardiology/ 4/4
7/31/2017 Cardiology - FRCEM Success

Dashboard Subscription expires in: 82 Days Extend

Question Navigator
Cardiology
Question 2 of 24 1 Unanswered

An 81 year old lady is brought in by ambulance acutely breathless and distressed. She has a 2 Current Question
history of chronic heart failure, hypertension and diabetes. Her observations are: BP 110/85, HR
3 Unanswered
110 bpm, RR 26 breaths/min, and sats 97% on 15 L oxygen through a reservoir mask. Her chest
x-ray shows orid pulmonary oedema. 4 Unanswered

a. Give two drugs (including dose and route) that could be given initially to ease this 5 Unanswered
patient’s symptoms. (1 mark)
6 Unanswered
b. Despite medical therapy, after an hour the patient remains breathless and hypoxic. An
arterial blood gas is performed which shows persistent type I respiratory failure. What 7 Unanswered
additional therapy could you initiate in AED and how would you initiate this therapy? (1
8 Unanswered
mark)
c. Give three possible causes of non-cardiogenic pulmonary oedema. (1 mark) 9 Unanswered

10 Unanswered

You did not answer this question 11 Unanswered

12 Unanswered

Answer
a. Any two of:
http://intermediate.frcemsuccess.com/rev/cardiology/ 1/4
7/31/2017 Cardiology - FRCEM Success

GTN (1 – 2 doses of sublingual 400 microgram spray or 300 microgram tablet)


IV furosemide 40 – 80 mg
IV diamorphine 2.5 – 5 mg
b. Non-invasive ventilation – continuous positive airway pressure (CPAP) – start at approx. 5 cm H2O (and
increase to maximum of 15 cm H2O as tolerated)
c. Any two of:
N: near drowning
O: O2 therapy/post intubation pulmonary oedema
T: transfusion (TRALI: transfusion-related acute lung injury)
C: CNS: neurogenic pulmonary oedema e.g. intracranial haemorrhage
A: allergic alveolitis
A: adult respiratory distress syndrome (ARDs) e.g. sepsis, pancreatitis, trauma
R: renal failure
D: drugs
I: inhaled (toxins) e.g. smoke
A: altitude: high altitude pulmonary oedema (HAPE)
C: contusion

Notes

Acute cardiogenic pulmonary oedema:

Causes:

Acute deterioration of chronic heart failure


Sudden-onset cardiac arrhythmia
Ischaemic heart disease
Acute valvular dysfunction e.g. mitral stenosis
Cardiac tamponade
Cardiomyopathy
Negative inotropes e.g. beta-blockers
Acute myocarditis
Acute pericarditis
Fluid overload

http://intermediate.frcemsuccess.com/rev/cardiology/ 2/4
7/31/2017 Cardiology - FRCEM Success

Clinical features:

Symptoms
Breathlessness
Orthopnoea
Paroxysmal nocturnal dyspnoea
Coughing (rarely, with frothy blood-stained sputum)
Signs
Respiratory distress
Cyanosis
Tachycardia
Elevated JVP
Gallop rhythm
Inspiratory crackles at lung bases
Cardiac wheeze
Peripherally shut down (peripherally cool, pale, increased CRT)

Investigations:

CXR
Upper lobe diversion (distension of upper pulmonary veins)
Kerley B septal lines ( uid in the interstitium)
Bat’s wing hilar shadowing (blurred margins of the hilar vessels)
Fluid in interlobar ssures
Interstitial shadowing
Alveolar shadowing in orid oedema
Pleural effusions
Cardiomegaly
ECG
Arrhythmia
Ischaemia
Signs of heart failure e.g. ventricular strain pattern

Management:

Sit patient upright


http://intermediate.frcemsuccess.com/rev/cardiology/ 3/4
7/31/2017 Cardiology - FRCEM Success
Sit patient upright
Oxygen therapy to maintain saturations of 94 – 98% (or 88 – 92% in patients at risk of hypercapnic
respiratory failure)
Intravenous diuretic e.g. furosemide 40 – 80 mg, given slowly
Intravenous opioid e.g. diamorphine 2.5 – 5 mg, given slowly over 5 minutes
Intravenous antiemetic e.g. metoclopramide 10 mg
Sublingual/buccal nitrate e.g. GTN spray followed by nitrate infusion starting at 10 mcg/min and titrating
according to clinical response (as long as systolic BP remains > 90 mmHg)
Consider catheter and monitor urine output
Non-invasive ventilation (CPAP)
If hypotensive/shocked, may require inotropes and HDU care

< Previous Next > Submit Something wrong?

FRCEM Success Resources


We are an online revision resource for The Royal College of Emergency Advanced Life Support Group
FRCEM Primary and Intermediate Medicine Emergency Medicine Journal
exam preparation. Irish Association for Emergency Lifeinthefastlane
Medicine Instant Anatomy
Terms & Conditions Advanced Trauma Life Support Patient.co.uk
Get in Touch Resuscitation Council (UK)
TeachMeAnatomy
Trauma.org
Radiopaedia

©2014 - 2017 FRCEM Success | Website designed & hosted by Cyberfrog Design

http://intermediate.frcemsuccess.com/rev/cardiology/ 4/4
7/31/2017 Cardiology - FRCEM Success

Dashboard Subscription expires in: 82 Days Extend

Question Navigator
Cardiology
Question 3 of 24 1 Unanswered

A 71 year old man, with a history of hypertension and diabetes, presents to AED 2 Unanswered
complaining of central crushing chest pain that has been ongoing for about 25 minutes. His
3 Current Question
observations are: BP 145/80, HR 105, RR 18, sats 96 % OA. You suspect acute coronary
syndrome (ACS). 4 Unanswered

a. Give four drugs (including dose and route) that should initially be given to this patient. (1 5 Unanswered
mark)
6 Unanswered
b. What ECG ndings would indicate a non ST-elevation myocardial infarction (NSTEMI)?
(1 mark) 7 Unanswered
c. Other than ACS, give one further cardiac and one non-cardiac cause for a raised
8 Unanswered
troponin. (1 mark)
9 Unanswered

10 Unanswered
You did not answer this question
11 Unanswered

12 Unanswered
Answer
a. All four of (although reasonable alternatives for analgesia/antiemetic may be used):
Sublingual GTN – 1 – 2 doses of sublingual 400 microgram spray or 300 microgram tablet
http://intermediate.frcemsuccess.com/rev/cardiology/ 1/4
7/31/2017 Cardiology - FRCEM Success

Analgesia e.g. IV diamorphine 2.5 – 5 mg


Antiemetic e.g. IV metoclopramide 10 mg
Aspirin 300 mg od
b. Regional ST-depression or deep T-wave inversion
c. Any two of:
Cardiac
Arrhythmias
Pericarditis
Myocarditis
Aortic dissection
Heart failure
Cardiac surgery/intervention
Heart transplant
Cardiac contusion in blunt wall trauma
Cardiac in ltrative disorders e.g. amyloidosis
Non-cardiac
Chemotherapy
Sepsis
Rhabdomyolysis
End-stage renal disease
Strenuous exercise
Pulmonary embolus

Notes

Management of ACS:

If ACS is suspected:

Oxygen to maintain saturations of 94 – 98% (or 88 – 92% in patients at risk of hypercapnic respiratory
failure)
GTN (1 – 2 doses of sublingual 400 microgram spray or 300 microgram tablet)
Analgesia e.g. IV diamorphine 2.5 – 5 mg, given slowly over 5 minutes
Antiemetic e.g. IV metoclopramide 10 mg
Aspirin 300 mg (unless there is clear evidence of allergy)
http://intermediate.frcemsuccess.com/rev/cardiology/ 2/4
7/31/2017 Cardiology - FRCEM Success
Aspirin 300 mg (unless there is clear evidence of allergy)
If pain is ongoing, consider intravenous GTN (10 – 200 micrograms/min) or ISM (2 – 10 mg/hr) IV infusion

For diagnosed NSTEMI/Unstable angina (in addition to aspirin 300 mg):

Fondaparinux (2.5 mg SC) for patients without a high bleeding risk, unless coronary angiography is planned
within 24 hours
Unfractionated heparin is an alternative to fondaparinux if angiography is planned within 24 hours of
admission (or for patients with signi cant renal impairment)
For patients at low risk (mortality < 3%) offer 300 mg loading dose of clopidogrel only to those who may
undergo PCI within 24 h of admission to hospital
For patients at higher risk (mortality > 3%) offer 300 mg loading dose of clopidogrel to all patients; consider
adding a GPI (intravenous epti batide or tiro ban) if angiography is scheduled within 96 hours of hospital
admission
ADP receptor blocker alternatives to clopidogrel 300 mg include prasugrel 60 mg or ticagrelor 180 mg
Additional drug therapy may include beta-blockers (diltiazem is alternative), ACE inhibitors and statins

For diagnosed STEMI (in addition to aspirin 300 mg):

Start antiplatelets/antithrombotics as per local protocol


For patients undergoing PPCI:
All patients undergoing PPCI should receive a platelet ADP receptor blocker e.g. clopidogrel 600
mg, ticagrelor 180 mg or prasugrel 60 mg
Anticoagulation with unfractionated or LMWH is given in the catheter lab, and in high risk cases a
glycoprotein IIb/IIIa inhibitor may also be given. Bivalirudin, a direct thrombin inhibitor may be
chosen as an alternative to heparin
For patients undergoing brinolysis:
Patients undergoing brinolysis should receive: clopidogrel 300 mg od and antithrombin therapy
(unfractionated heparin, LMWH or fondaparinux)
Offer coronary angiography, with follow-on primary PCI if indicated, as the preferred coronary reperfusion
strategy for people with acute STEMI if presentation is within 12 hours of onset of symptoms AND primary
PCI can be delivered within 120 minutes of the time when brinolysis could have been given
Consider coronary angiography, with follow-on primary PCI if indicated, for people with acute STEMI
presenting more than 12 hours after the onset of symptoms if there is evidence of continuing myocardial
ischaemia or of cardiogenic shock
For those who undergo brinolysis (indicated in those presenting within 12 hours of onset of symptoms in
http://intermediate.frcemsuccess.com/rev/cardiology/ 3/4
7/31/2017 Cardiology - FRCEM Success

whom primary PCI cannot be delivered in a reasonable timeframe), repeat an ECG 60 – 90 minutes after
administration. For those with residual ST-segment elevation, offer immediate coronary angiography, with
follow-on rescue PCI, if indicated

< Previous Next > Submit Something wrong?

FRCEM Success Resources


We are an online revision resource for The Royal College of Emergency Advanced Life Support Group
FRCEM Primary and Intermediate Medicine Emergency Medicine Journal
exam preparation. Irish Association for Emergency Lifeinthefastlane
Medicine Instant Anatomy
Terms & Conditions Advanced Trauma Life Support Patient.co.uk
Get in Touch Resuscitation Council (UK)
TeachMeAnatomy
Trauma.org
Radiopaedia

©2014 - 2017 FRCEM Success | Website designed & hosted by Cyberfrog Design

http://intermediate.frcemsuccess.com/rev/cardiology/ 4/4
7/31/2017 Cardiology - FRCEM Success

Dashboard Subscription expires in: 82 Days Extend

Question Navigator
Cardiology
Question 4 of 24 1 Unanswered

A 65 year old man, with a history of polycystic kidney disease, presents to AED complaining 2 Unanswered
of headache and blurred vision. His observations are: BP 220/115, HR 98 bpm, RR 16, temp
3 Unanswered
36.5°C.
4 Current Question
a. Give three drugs that you could give to this patient in AED to lower his BP. (1 mark)
b. What class of drug should be used initially if severe hypertension is secondary to 5 Unanswered
pheochromocytoma? (1 mark)
6 Unanswered
c. Calculate this patient’s mean arterial pressure (MAP). (1 mark)
7 Unanswered

8 Unanswered
You did not answer this question
9 Unanswered

10 Unanswered
Answer 11 Unanswered
a. Any three of:
12 Unanswered
Sodium nitroprusside
Nicardipine hydrochloride
Labetalol hydrochloride
Glyceryl trinitrate
http://intermediate.frcemsuccess.com/rev/cardiology/ 1/4
7/31/2017 Cardiology - FRCEM Success

Phentolamine mesylate (drug of choice for pheochromocytoma crisis)


Hydralazine hydrochloride
Esmolol hydrochloride
b. Alpha-blocker e.g. Phentolamine
c. MAP = 150 = [(2 x DBP) +SBP)]/3 = [230 + 220]/3 = 450/3

Notes

Severe hypertension (blood pressure ≥ 180/110 mmHg) without acute target-organ damage is de ned as a
hypertensive urgency; blood pressure should be reduced gradually over 24 – 48 hours with oral antihypertensive
therapy, such as labetalol hydrochloride, or the calcium-channel blockers amlodipine or felodipine.

A hypertensive emergency is de ned as severe hypertension with acute damage to the target organs (e.g. signs of
papilloedema or retinal haemorrhage, or the presence of clinical conditions such as acute coronary syndromes,
acute aortic dissection, acute pulmonary oedema, hypertensive encephalopathy, acute cerebral infarction,
intracerebral or subarachnoid haemorrhage, eclampsia, or rapidly progressing renal failure). If blood pressure is
reduced too quickly in the management of hypertensive crises, there is a risk of reduced organ perfusion leading to
cerebral infarction, blindness, deterioration in renal function, and myocardial ischaemia.

Clinical features:

Asymptomatic
Headache
Nausea and vomiting
Visual disturbance
Chest pain
Seizures
Stroke
Retinopathy with papilloedema

Management of hypertensive crisis:

Prompt treatment with intravenous antihypertensive therapy is generally required


Over the rst few minutes or within 2 hours, blood pressure should be reduced by 20 – 25%
When intravenous therapy is indicated, treatment options include:
Sodium nitroprusside
http://intermediate.frcemsuccess.com/rev/cardiology/ 2/4
7/31/2017 Cardiology - FRCEM Success
Sodium nitroprusside
Nicardipine hydrochloride
Labetalol hydrochloride
Glyceryl trinitrate
Phentolamine mesylate (drug of choice for pheochromocytoma crisis)
Hydralazine hydrochloride
Esmolol hydrochloride

Causes of secondary hypertension:

Renal
Renal artery stenosis
Polycystic kidneys
Intrinsic renal disease
Endocrine
Cushing’s syndrome
Conn’s syndrome
Pheochromocytoma
Acromegaly
Hyperparathyroidism
Drugs e.g. alcohol, cocaine
Pregnancy and pre-eclampsia
Coarctation of the aorta

< Previous Next > Submit Something wrong?

FRCEM Success Resources


We are an online revision resource for The Royal College of Emergency Advanced Life Support Group
FRCEM Primary and Intermediate Medicine Emergency Medicine Journal
exam preparation. Irish Association for Emergency Lifeinthefastlane
Medicine Instant Anatomy

http://intermediate.frcemsuccess.com/rev/cardiology/ 3/4
7/31/2017 Cardiology - FRCEM Success

Dashboard Subscription expires in: 82 Days Extend

Question Navigator
Cardiology
Question 5 of 24 1 Unanswered

A 45 year old man, known to inject drugs, and currently living in homeless hostels, presents 2 Unanswered
to AED complaining of malaise, fever, rigors and arthralgia. On auscultation he has a new
3 Unanswered
pansystolic murmur heard loudest over the left sternal edge.
4 Unanswered
a. What diagnosis should be considered in this patient? (1 mark)
b. Give two clinical features you may see in the hands of a patient with this condition. (1 5 Current Question
mark)
6 Unanswered
c. Give two investigations that are most useful to make this diagnosis. (1 mark)
7 Unanswered

8 Unanswered
You did not answer this question
9 Unanswered

10 Unanswered
Answer 11 Unanswered
a. Infective endocarditis
12 Unanswered
b. Any two of:
Splinter (subungual) haemorrhages
Clubbing
Osler nodes (small tender red-to-purple nodules on the pulp of the terminal phalanges of the ngers
http://intermediate.frcemsuccess.com/rev/cardiology/ 1/3
7/31/2017 Cardiology - FRCEM Success

and toes)
Janeway’s lesions (irregular painless erythematous macules on the thenar and hypothenar eminence)
c. Blood cultures (x 3) and transthoracic (or transoesophageal) echo

Notes

Infective endocarditis:

Risk factors:

Valvular heart disease


Prosthetic valves
Structural congenital heart disease
Rheumatic heart disease
Previous infective endocarditis
Hypertrophic cardiomyopathy
Intravenous drug use (IVDU)
Invasive vascular procedures

Common organisms:

Staphylococcus aureus (most common cause overall – acute and subacute IE)
Streptococcus viridans (most common cause of subacute IE)
Pseudomonas aeruginosa
HACEK (Haemophilus spp.,Aggregatibacter actinomycetemcomitans, Cardiobacterium spp., Eikenella
corrodens, Kingella kingae)

Clinical features:

Fever
New heart murmur
Non-speci c systemic features e.g. malaise, anorexia, weight loss, sweats, myalgia
Arthritis
Splenomegaly
Signs
Splinter (subungual) haemorrhages
http://intermediate.frcemsuccess.com/rev/cardiology/ 2/3
7/31/2017 Cardiology - FRCEM Success
Splinter (subungual) haemorrhages
Clubbing
Roth’s spots (retinal haemorrhages with pale centres)
Osler nodes (small tender red-to-purple nodules on the pulp of the terminal phalanges of the ngers
and toes)
Janeway’s lesions (irregular painless erythematous macules on the thenar and hypothenar eminence)

Investigations:

Echo – initially transthoracic, TOE is more sensitive


Blood cultures (x 3 at different sites and times, and preferably during fever)
FBC/CRP (nonspeci c raised in ammatory markers)

Management:

Empirical antibiotics according to local protocol


Refer urgently to cardiology

< Previous Next > Submit Something wrong?

FRCEM Success Resources


We are an online revision resource for The Royal College of Emergency Advanced Life Support Group
FRCEM Primary and Intermediate Medicine Emergency Medicine Journal
exam preparation. Irish Association for Emergency Lifeinthefastlane
Medicine Instant Anatomy
Terms & Conditions Advanced Trauma Life Support Patient.co.uk
Get in Touch Resuscitation Council (UK)
TeachMeAnatomy
Trauma.org
Radiopaedia

©2014 - 2017 FRCEM Success | Website designed & hosted by Cyberfrog Design

http://intermediate.frcemsuccess.com/rev/cardiology/ 3/3
7/31/2017 Cardiology - FRCEM Success

Dashboard Subscription expires in: 82 Days Extend

Question Navigator
Cardiology
Question 6 of 24 1 Unanswered

An 81 year old lady is brought in by ambulance acutely breathless and distressed. She has a 2 Unanswered
history of chronic heart failure, hypertension and diabetes. Her observations are: BP 110/85, HR
3 Unanswered
110 bpm, RR 26 breaths/min, and sats 97% on 15 L oxygen through a reservoir mask. A mobile
chest x-ray is performed in Resus which is shown below: 4 Unanswered

5 Unanswered

6 Current Question

7 Unanswered

8 Unanswered

9 Unanswered

10 Unanswered

11 Unanswered

12 Unanswered

http://intermediate.frcemsuccess.com/rev/cardiology/ 1/5
7/31/2017 Cardiology - FRCEM Success

Courtesy of Frank Gaillard [GFDL 1.3 (www.gnu.org/licenses/fdl-1.3.html), GFDL 1.3


(www.gnu.org/licenses/fdl-1.3.html) or CC BY-SA 3.0 (http://creativecommons.org/licenses/by-
sa/3.0)], via Wikimedia Commons

a. Give two abnormalities seen on the chest x-ray and give the radiological diagnosis. (1
mark)
b. Give three ndings you might expect to see on examination of this patient. (1 mark)
c. Give two factors that may have precipitated the clinical picture in this patient. (1 mark)

You did not answer this question

Answer
a. Pulmonary oedema – Any two of:
Alveolar shadowing
Interstitial shadowing
Kerley B septal lines
Peribronchial cuf ng
Blunting of right costophrenic angle
b. Any three of:
Tachycardia
Elevated JVP
http://intermediate.frcemsuccess.com/rev/cardiology/ 2/5
7/31/2017 Cardiology - FRCEM Success
Elevated JVP
Gallop rhythm
Inspiratory crackles
Cardiac wheeze
Cool, pale peripheries, CRT > 2 secs
c. Any two of:
Non-compliance with medications
New ischaemia/ACS
New arrhythmia – commonly atrial brillation
New valvular disease
Worsening comorbidities e.g. anaemia, hyperthyroidism, hypertension, pulmonary disease, diabetes,
renal disease
Infection
Electrolyte imbalance
Fluid overload
New medication/Change in dose of medication e.g. negative inotropes

Notes

Acute cardiogenic pulmonary oedema:

Causes:

Acute deterioration of chronic heart failure


Sudden-onset cardiac arrhythmia
Ischaemic heart disease
Acute valvular dysfunction e.g. mitral stenosis
Cardiac tamponade
Cardiomyopathy
Negative inotropes e.g. beta-blockers
Acute myocarditis
Acute pericarditis
Fluid overload

Clinical features:
http://intermediate.frcemsuccess.com/rev/cardiology/ 3/5
7/31/2017 Cardiology - FRCEM Success

Symptoms
Breathlessness
Orthopnoea
Paroxysmal nocturnal dyspnoea
Coughing (rarely, with frothy blood-stained sputum)
Signs
Respiratory distress
Cyanosis
Tachycardia
Elevated JVP
Gallop rhythm
Inspiratory crackles at lung bases
Cardiac wheeze
Peripherally shut down (peripherally cool, pale, increased CRT)

Investigations:

CXR
Upper lobe diversion (distension of upper pulmonary veins)
Kerley B septal lines ( uid in the interstitium)
Bat’s wing hilar shadowing (blurred margins of the hilar vessels)
Fluid in interlobar ssures
Interstitial shadowing
Alveolar shadowing in orid oedema
Pleural effusions
Cardiomegaly
ECG
Arrhythmia
Ischaemia
Signs of heart failure e.g. ventricular strain pattern

Management

Sit patient upright


Oxygen therapy to maintain saturations of 94 – 98% (or 88 – 92% in patients at risk of hypercapnic
http://intermediate.frcemsuccess.com/rev/cardiology/ 4/5
7/31/2017 Cardiology - FRCEM Success
Oxygen therapy to maintain saturations of 94 – 98% (or 88 – 92% in patients at risk of hypercapnic
respiratory failure)
Intravenous diuretic e.g. furosemide 40 – 80 mg, given slowly
Intravenous opioid e.g. diamorphine 2.5 – 5 mg, given slowly over 5 minutes
Intravenous antiemetic e.g. metoclopramide 10 mg
Sublingual/buccal nitrate e.g. GTN spray followed by nitrate infusion starting at 10 mcg/min and titrating
according to clinical response (as long as systolic BP remains > 90 mmHg)
Consider catheter and monitor urine output
Non-invasive ventilation (CPAP)
If hypotensive/shocked, may require inotropes and HDU care

< Previous Next > Submit Something wrong?

FRCEM Success Resources


We are an online revision resource for The Royal College of Emergency Advanced Life Support Group
FRCEM Primary and Intermediate Medicine Emergency Medicine Journal
exam preparation. Irish Association for Emergency Lifeinthefastlane
Medicine Instant Anatomy
Terms & Conditions Advanced Trauma Life Support Patient.co.uk
Get in Touch Resuscitation Council (UK)
TeachMeAnatomy
Trauma.org
Radiopaedia

©2014 - 2017 FRCEM Success | Website designed & hosted by Cyberfrog Design

http://intermediate.frcemsuccess.com/rev/cardiology/ 5/5
7/31/2017 Cardiology - FRCEM Success

Dashboard Subscription expires in: 82 Days Extend

Question Navigator
Cardiology
Question 7 of 24 1 Unanswered

An 81 year old lady is brought in by ambulance acutely breathless and distressed. She has a 2 Unanswered
history of chronic heart failure, hypertension and diabetes. Her observations are: BP 110/85, HR
3 Unanswered
110 bpm, RR 26 breaths/min, and sats 97% on 15 L oxygen through a reservoir mask. Her chest
x-ray shows orid oedema. 4 Unanswered

a. You examine the patient and assess her JVP. Give two features of the JVP that allow you 5 Unanswered
to differentiate it from the carotid pulse.(1 mark)
6 Unanswered
b. Other than heart failure, give two additional causes of a raised JVP. (1 mark)
c. What two important investigations should be performed in this patient and give your 7 Current Question
rationale? (1 mark)
8 Unanswered

9 Unanswered
You did not answer this question
10 Unanswered

11 Unanswered

Answer 12 Unanswered

a. Any two of:


Double waveform
Not palpable
http://intermediate.frcemsuccess.com/rev/cardiology/ 1/4
7/31/2017 Cardiology - FRCEM Success

Occludable
Variable with respiration (decreases with inspiration)
Enhanced by the hepatojugular re ux manoeuvre
b. Any two of:
Fluid overload e.g. excessive uid administration, renal failure
Constrictive pericarditis
Cardiac tamponade
Tricuspid stenosis
Superior vena cava obstruction
Tension pneumothorax
Pulmonary embolism
High output states
Complete heart block
c. ECG and serum troponin to exclude ACS that may have precipitated acute heart failure in this patient

Notes

Acute cardiogenic pulmonary oedema:

Causes:

Acute deterioration of chronic heart failure


Sudden-onset cardiac arrhythmia
Ischaemic heart disease
Acute valvular dysfunction e.g. mitral stenosis
Cardiac tamponade
Cardiomyopathy
Negative inotropes e.g. beta-blockers
Acute myocarditis
Acute pericarditis
Fluid overload

Clinical features:

Symptoms
http://intermediate.frcemsuccess.com/rev/cardiology/ 2/4
7/31/2017 Cardiology - FRCEM Success

Breathlessness
Orthopnoea
Paroxysmal nocturnal dyspnoea
Coughing (rarely, with frothy blood-stained sputum)
Signs
Respiratory distress
Cyanosis
Tachycardia
Elevated JVP
Gallop rhythm
Inspiratory crackles at lung bases
Cardiac wheeze
Peripherally shut down (peripherally cool, pale, increased CRT)

Investigations:

CXR
Upper lobe diversion (distension of upper pulmonary veins)
Kerley B septal lines ( uid in the interstitium)
Bat’s wing hilar shadowing (blurred margins of the hilar vessels)
Fluid in interlobar ssures
Interstitial shadowing
Alveolar shadowing in orid oedema
Pleural effusions
Cardiomegaly
ECG
Arrhythmia
Ischaemia
Signs of heart failure e.g. ventricular strain pattern

Management:

Sit patient upright


Oxygen therapy to maintain saturations of 94 – 98% (or 88 – 92% in patients at risk of hypercapnic
respiratory failure)
Intravenous diuretic e.g. furosemide 40 – 80 mg, given slowly
http://intermediate.frcemsuccess.com/rev/cardiology/ 3/4
7/31/2017 Cardiology - FRCEM Success
Intravenous diuretic e.g. furosemide 40 – 80 mg, given slowly
Intravenous opioid e.g. diamorphine 2.5 – 5 mg, given slowly over 5 minutes
Intravenous antiemetic e.g. metoclopramide 10 mg
Sublingual/buccal nitrate e.g. GTN spray followed by nitrate infusion starting at 10 mcg/min and titrating
according to clinical response (as long as systolic BP remains > 90 mmHg)
Consider catheter and monitor urine output
Non-invasive ventilation (CPAP)
If hypotensive/shocked, may require inotropes and HDU care

< Previous Next > Submit Something wrong?

FRCEM Success Resources


We are an online revision resource for The Royal College of Emergency Advanced Life Support Group
FRCEM Primary and Intermediate Medicine Emergency Medicine Journal
exam preparation. Irish Association for Emergency Lifeinthefastlane
Medicine Instant Anatomy
Terms & Conditions Advanced Trauma Life Support Patient.co.uk
Get in Touch Resuscitation Council (UK)
TeachMeAnatomy
Trauma.org
Radiopaedia

©2014 - 2017 FRCEM Success | Website designed & hosted by Cyberfrog Design

http://intermediate.frcemsuccess.com/rev/cardiology/ 4/4
7/31/2017 Cardiology - FRCEM Success

Dashboard Subscription expires in: 82 Days Extend

Question Navigator
Cardiology
Question 8 of 24 1 Unanswered

A 65 year old man presents to AED with sudden onset ‘ripping’ chest pain that radiates 2 Unanswered
through to his back. The pain is severe and was maximal at onset. His observations are: BP
3 Unanswered
208/110, HR 105 bpm, RR 20, sats 96% OA.
4 Unanswered
a. What serious diagnosis should be considered in this patient? (1 mark)
b. Give two risk factors for this condition. (1 mark) 5 Unanswered
c. What is the most important step of initially managing this patient in the AED
6 Unanswered
department, giving your rationale? (1 mark)
7 Unanswered

8 Current Question
You did not answer this question
9 Unanswered

10 Unanswered
Answer 11 Unanswered
a. Aortic dissection
12 Unanswered
b. Any two of:
Hypertension
Pre-existing aortic or aortic valve disease
Family history of aortic disease
http://intermediate.frcemsuccess.com/rev/cardiology/ 1/4
7/31/2017 Cardiology - FRCEM Success

Connective tissue disease (e.g. Marfan’s syndrome, Ehlers-Danlos syndrome)


Smoking
History of cardiac surgery
Direct blunt chest trauma
c. Lower his BP – high blood pressure drives progression of dissection

Notes

Aortic dissection is de ned as disruption of the medial layer of the aortic wall provoked by intramural bleeding,
resulting in separation of the aortic wall layers and subsequent formation of a true lumen and a false lumen, with or
without communication.

Risk factors:

Hypertension
Pre-existing aortic or aortic valve disease
Family history of aortic disease
Connective tissue disease (e.g. Marfan’s syndrome, Ehlers-Danlos syndrome)
Smoking
History of cardiac surgery
Direct blunt chest trauma

Clinical features:

Sudden tearing/ripping/sharp chest pain, maximal at onset, radiating to the back and inter-scapular area
Syncope
Hypertension (although hypotension may be present in tamponade)
Asymmetrical blood pressures in each arms
Inequality in pulses on each side
New diastolic murmur (aortic regurgitation)
Pericardial friction rub
Neurological de cit

Dissection may be classi ed as Stanford type A or B:

Type A involves the ascending aorta and/or aortic arch (35% mortality) – usually treated surgically
http://intermediate.frcemsuccess.com/rev/cardiology/ 2/4
7/31/2017 Cardiology - FRCEM Success
Type A involves the ascending aorta and/or aortic arch (35% mortality) – usually treated surgically
Type B involves only the descending aorta and arises distal to the origin of the left subclavian artery (15%
mortality) – usually treated medically

Investigations:

CXR
Widened/abnormal mediastinum
Double knuckle aorta
Left pleural effusion
Deviation of the trachea to the right
Separation of two parts of the wall of a calci ed aorta by > 5 mm
ECG – may show ischaemia as a consequence of aortic dissection
Echo
CT aorta – de nitive diagnosis

Complications:

Acute coronary ischaemia


Neurological de cit
Limb ischaemia
Cardiac tamponade
Aortic regurgitation
Haemothorax
Stroke
Aortic-oesophageal stula
Retroperitoneal haemorrhage
Mesenteric ischaemia
Renal failure

< Previous Next > Submit Something wrong?

http://intermediate.frcemsuccess.com/rev/cardiology/ 3/4
7/31/2017 Cardiology - FRCEM Success

Dashboard Subscription expires in: 82 Days Extend

Question Navigator
Cardiology
Question 9 of 24 1 Unanswered

A 61 year old woman, with a history of hypertension, presents to AED complaining of 2 Unanswered
sudden onset palpitations which came on about 2 hours ago. Her observations are: BP 125/95,
3 Unanswered
HR 150 bpm, RR 20, sats 98% OA, temp 36.7°C. Her ECG is shown below:
4 Unanswered

5 Unanswered

6 Unanswered

7 Unanswered

8 Unanswered

9 Current Question

10 Unanswered

11 Unanswered

12 Unanswered

Courtesy of James Heilman, MD (Own work) [CC BY-SA 3.0 (http://creativecommons.org/licenses/by-


sa/3.0)], via Wikimedia Commons

http://intermediate.frcemsuccess.com/rev/cardiology/ 1/5
7/31/2017 Cardiology - FRCEM Success

a. Other than hypertension, give three factors which may have precipitated this condition.
(1 mark)
b. Give a drug (including dose and route) that could be used rst-line for rate-control. (1
mark)
c. The patient suddenly develops central chest pain radiating down her arm. A repeat ECG
shows ischaemic change. What treatment is now indicated? (1 mark)

You did not answer this question

Answer
a. Any three of:
Ischaemic heart disease
Valvular heart disease
Hyperthyroidism
Rheumatic heart disease
Heart failure
Structural cardiac disease
Lung cancer
Drugs e.g. thyroxine, beta-agonists
Acute infection
Electrolyte imbalance
Pulmonary embolism
Excessive caffeine/alcohol
Diabetes mellitus
b. Any one of:
Atenolol 5 mg IV (repeated if necessary after 10 minutes)
Metoprolol 2 – 5 mg IV (at 5 minute intervals to a total of 15 mg)
Bisoprolol 5 mg PO
Propranolol 100 micrograms IV slowly in three equal doses at 2 -3 minute intervals
http://intermediate.frcemsuccess.com/rev/cardiology/ 2/5
7/31/2017 Cardiology - FRCEM Success
Propranolol 100 micrograms IV slowly in three equal doses at 2 -3 minute intervals
Esmolol 500 micrograms IV loading dose over 1 minute, followed by an infusion of 50 – 200
micrograms/kg/min
c. Electrical synchronised DC cardioversion

Notes

Atrial brillation results from irregular, disorganised electrical activity in the atria, causing disorganised atrial
depolarisation and ineffective atrial contractions. The atrioventricular node receives more electrical impulses than
it can conduct, resulting in an irregular ventricular rhythm.

Most common causes:

Idiopathic
Ischaemic heart disease
Hypertension
Valvular heart disease
Hyperthyroidism

Clinical features:

Asymptomatic
Dyspnoea
Palpitations
Chest pain
Syncope/dizziness
Reduced exercise tolerance

Investigations:

ECG
Absent P-waves
Irregularly irregular ventricular rate (may be tachycardic)
Ambulatory 24-hour ECG
Bloods
FBC, CRP, U&Es, TFTs, LFTs, calcium, magnesium, glucose
Echo if suspicion of underlying structural heart disease
http://intermediate.frcemsuccess.com/rev/cardiology/ 3/5
7/31/2017 Cardiology - FRCEM Success
Echo if suspicion of underlying structural heart disease
CXR if lung pathology is suspected

Management:

Identify and manage any underlying causes or triggers of AF


A rate-control treatment is recommended for most people with AF
Beta-blocker e.g. metoprolol (5 mg IV) or bisoprolol (5 mg PO)
Rate-limiting calcium-channel blocker e.g. diltiazem or verapamil (5 mg IV); if beta-blockers are
contraindicated or not tolerated
Digoxin 500 micrograms IV over 30 mins (only recommended for predominantly sedentary patients, for
patients in heart failure or in addition to other treatments where monotherapy is unsuccessful)
Rhythm control (electrical or pharmacological cardioversion) may be appropriate for people:
With new-onset AF (< 48 hours of onset)
Whose AF has a reversible cause (for example a chest infection)
Who have heart failure thought to be primarily caused, or worsened, by AF
With atrial utter and is considered suitable for an ablation strategy to restore sinus rhythm
For whom a rhythm control strategy would be more suitable based on clinical judgement
For cardioversion in patients with AF with onset < 48 hrs
Emergency electrical cardioversion in people with life-threatening haemodynamic instability caused by
new-onset AF
Pharmacological
amiodarone (300 mg IV) in patients with evidence of structural heart disease,
ecainide (50 – 150 mg IV) in patients with no evidence of structural/ischaemic heart disease
If cardioversion is being considered in AF with onset > 48 hrs, it should be delayed until the person has been
maintained on therapeutic anticoagulation for a minimum of 3 weeks (and electrical cardioversion is
preferred)
Assess stroke risk using the CHA2DS2VASc assessment tool
Offer anticoagulation to all people with a CHA2DS2VASc score of 2 or above, and consider offering it
to men with a CHA2DS2VASc score of 1, after taking into account the person’s bleeding risk
Assess risk of bleeding using the HAS-BLED assessment tool
Anticoagulant options include warfarin and the new agents dabigatran, rivaroxaban or apixaban

< Previous Next > Submit Something wrong?


http://intermediate.frcemsuccess.com/rev/cardiology/ 4/5
7/31/2017 Cardiology - FRCEM Success

Dashboard Subscription expires in: 82 Days Extend

Question Navigator
Cardiology
Question 10 of 24 1 Unanswered

A 65 year old man, with a history of polycystic kidney disease, presents to AED complaining 2 Unanswered
of headache and blurred vision. His observations are: BP 220/115, HR 98 bpm, RR 16, temp
3 Unanswered
36.5°C.
4 Unanswered
a. De ne ‘severe hypertension’, and a ‘hypertensive emergency’. (1 mark)
b. Give three possible consequences of severe hypertension. (1 mark) 5 Unanswered
c. Give two further causes of secondary hypertension. (1 mark)
6 Unanswered

7 Unanswered
You did not answer this question
8 Unanswered

9 Unanswered

Answer 10 Current Question

a. Severe hypertension = SBP > 180 or DBP > 110. Hypertensive emergency = severe hypertension with acute 11 Unanswered
damage of target organs
12 Unanswered
b. Any three of:
Retinopathy
Acute coronary syndrome
Acute aortic dissection
http://intermediate.frcemsuccess.com/rev/cardiology/ 1/4
7/31/2017 Cardiology - FRCEM Success

Acute pulmonary oedema


Hypertensive encephalopathy
Acute cerebral infarction
Intracerebral or subarachnoid haemorrhage
Eclampsia
Rapidly progressive renal failure
c. Any two of:
Cushing’s syndrome
Conn’s syndrome
Pheochromocytoma
Acromegaly
Hyperparathyroidism
Drugs e.g. alcohol, cocaine
Pregnancy and pre-eclampsia
Coarctation of the aorta
Renal artery stenosis
Polycystic kidneys
Intrinsic renal disease

Notes

Severe hypertension (blood pressure ≥ 180/110 mmHg) without acute target-organ damage is de ned as a
hypertensive urgency; blood pressure should be reduced gradually over 24 – 48 hours with oral antihypertensive
therapy, such as labetalol hydrochloride, or the calcium-channel blockers amlodipine or felodipine.

A hypertensive emergency is de ned as severe hypertension with acute damage to the target organs (e.g. signs of
papilloedema or retinal haemorrhage, or the presence of clinical conditions such as acute coronary syndromes,
acute aortic dissection, acute pulmonary oedema, hypertensive encephalopathy, acute cerebral infarction,
intracerebral or subarachnoid haemorrhage, eclampsia, or rapidly progressing renal failure). If blood pressure is
reduced too quickly in the management of hypertensive crises, there is a risk of reduced organ perfusion leading to
cerebral infarction, blindness, deterioration in renal function, and myocardial ischaemia.

Clinical features:

Asymptomatic
http://intermediate.frcemsuccess.com/rev/cardiology/ 2/4
7/31/2017 Cardiology - FRCEM Success
Asymptomatic
Headache
Nausea and vomiting
Visual disturbance
Chest pain
Seizures
Stroke
Retinopathy with papilloedema

Management of hypertensive crisis:

Prompt treatment with intravenous antihypertensive therapy is generally required


Over the rst few minutes or within 2 hours, blood pressure should be reduced by 20 – 25%
When intravenous therapy is indicated, treatment options include:
Sodium nitroprusside
Nicardipine hydrochloride
Labetalol hydrochloride
Glyceryl trinitrate
Phentolamine mesylate (drug of choice for pheochromocytoma crisis)
Hydralazine hydrochloride
Esmolol hydrochloride

< Previous Next > Submit Something wrong?

FRCEM Success Resources


We are an online revision resource for The Royal College of Emergency Advanced Life Support Group
FRCEM Primary and Intermediate Medicine Emergency Medicine Journal
exam preparation. Irish Association for Emergency Lifeinthefastlane
Medicine Instant Anatomy
Terms & Conditions Advanced Trauma Life Support Patient.co.uk
Get in Touch Resuscitation Council (UK)
TeachMeAnatomy

http://intermediate.frcemsuccess.com/rev/cardiology/ 3/4
7/31/2017 Cardiology - FRCEM Success

Dashboard Subscription expires in: 82 Days Extend

Question Navigator
Cardiology
Question 11 of 24 1 Unanswered

A 39 year old woman presents to AED complaining of a sharp retrosternal chest pain which 2 Unanswered
has been ongoing for about 48 hours. The pain feels worse when she lies down at night, and
3 Unanswered
when she coughs or breathes deeply. An ECG has been performed by triage which is shown
below: 4 Unanswered

5 Unanswered

6 Unanswered

7 Unanswered

8 Unanswered

9 Unanswered

10 Unanswered

11 Current Question

12 Unanswered

Courtesy of James Heilman, MD (Own work) [CC BY-SA 3.0 (http://creativecommons.org/licenses/by-


http://intermediate.frcemsuccess.com/rev/cardiology/
sa/3.0)], via Wikimedia Commons 1/4
7/31/2017 Cardiology - FRCEM Success
sa/3.0)], via Wikimedia Commons

a. Give three abnormalities that can be seen on the ECG and give the likely diagnosis. (1
mark)
b. What classical clinical feature would you expect on examination of this patient? (1 mark)
c. Give two common viral causes for this condition. (1 mark)

You did not answer this question

Answer
a. Diagnosis = Acute pericarditis. Any three of:
Widespread concave (saddle-shaped) ST elevation in leads I, II, V2 – V6
ST depression in lead aVR
Tall peaked T waves
PR depression (best seen in lead II)
b. Pericardial friction rub
c. Any two of:
Coxsackie
Echovirus
Mumps
Epstein-Barr virus
Cytomegalovirus
In uenza
HIV

Notes

Acute pericarditis:

Causes:
http://intermediate.frcemsuccess.com/rev/cardiology/ 2/4
7/31/2017 Cardiology - FRCEM Success

Myocardial infarction (including Dressler’s syndrome)


Viral infection (e.g. Coxsackie, mumps, EBV, CMV, HIV)
Bacterial infection (e.g. Staphylococcus, Haemophilus)
Tuberculosis
Malignancy (e.g. bronchus or breast locally invading)
Rheumatic fever
Uraemia
Collagen vascular disease (e.g. SLE, RA)
Post-surgery, radiotherapy or trauma
Drugs (e.g. hydralazine, isoniazid, phenytoin, anticoagulants)

Clinical features:

Sharp, constant sternal pain relieved by sitting forwards, pain may radiate to the left shoulder and arm, and
is worse when lying on the left side and on inspiration, swallowing and coughing
Fever, cough, arthralgia
In cardiac tamponade – breathlessness, dysphagia, cough and hoarseness
Pericardial friction rub (high-pitched scratching sound, best heard over the left sternal border during
expiration)
In cardiac tamponade – pulsus paradoxus (decease in palpable pulse and arterial systolic BP of 10 mmHg on
inspiration), hypotension, muf ed heart sounds and jugular venous distension (Beck’s triad)

Investigations:

ECG
Sinus tachycardia
Widespread, saddle-shaped ST-elevation (in at least two limb leads and all chest leads)
ST-depression in leads aVR or I
PR depression
Tall, peaked T waves
QRS has decreased amplitude in pericardial effusion
CXR
Usually normal but may show a globular heart due to effusion, or associated pathology
Bloods
Troponin, FBC, CRP, U&E
http://intermediate.frcemsuccess.com/rev/cardiology/ 3/4
7/31/2017 Cardiology - FRCEM Success
Troponin, FBC, CRP, U&E
Echo
May demonstrate a pericardial effusion

Management is dependent on the cause to an extent but usually involves treatment with NSAIDs.

< Previous Next > Submit Something wrong?

FRCEM Success Resources


We are an online revision resource for The Royal College of Emergency Advanced Life Support Group
FRCEM Primary and Intermediate Medicine Emergency Medicine Journal
exam preparation. Irish Association for Emergency Lifeinthefastlane
Medicine Instant Anatomy
Terms & Conditions Advanced Trauma Life Support Patient.co.uk
Get in Touch Resuscitation Council (UK)
TeachMeAnatomy
Trauma.org
Radiopaedia

©2014 - 2017 FRCEM Success | Website designed & hosted by Cyberfrog Design

http://intermediate.frcemsuccess.com/rev/cardiology/ 4/4
7/31/2017 Cardiology - FRCEM Success

Dashboard Subscription expires in: 82 Days Extend

Question Navigator
Cardiology
Question 12 of 24 1 Unanswered

A 72 year old man with a history of hypertension and angina is brought to AED with severe 2 Unanswered
central chest pain radiating down his left arm that has not relieved with his usual GTN spray, and
3 Unanswered
has been ongoing now for about 1 hour. He has been given analgesia and an antiemetic. His
observations are: BP 148/85, HR 98, RR 17, sats 98% OA. His ECG is shown below: 4 Unanswered

5 Unanswered

6 Unanswered

7 Unanswered

8 Unanswered

9 Unanswered

10 Unanswered

11 Unanswered

12 Current Question

http://intermediate.frcemsuccess.com/rev/cardiology/ 1/5
7/31/2017 Cardiology - FRCEM Success

Courtesy of Displaced (Own work) [Public domain], via Wikimedia Commons

a. Describe the most obvious abnormalities seen on the ECG. (1 mark)


b. Give the ECG diagnosis and the most likely coronary vessel affected. (1 mark)
c. His pain continues despite receiving IV morphine. What drug (including dose and route)
should be started in this patient? (1 mark)

You did not answer this question

Answer
a. ST-elevation in leads V2 – V5, I, aVL AND reciprocal ST-depression in leads II, III, aVF
b. Anterolateral ST-elevation myocardial infarction (STEMI) – left anterior descending artery
c. GTN IV infusion (10 – 200 micrograms/min) or Isosorbide Mononitrate IV infusion (2 – 10 mg/hr)

Notes

In a patient with chest pain, acute coronary syndrome should be suspected if:

Pain in the chest or other areas (for example the arms, back, or jaw) lasts longer than 15 minutes
Chest pain is associated with nausea and vomiting, sweating or breathlessness, or a combination of these
Chest pain is associated with haemodynamic instability (for example the person has a systolic blood
pressure less than 90 mmHg)
Chest pain is of new-onset, or is the result of an abrupt deterioration of stable angina; with pain occurring
frequently with little or no exertion, and often lasting longer than 15 minutes
The patient’s response to GTN should not be used to con rm or exclude a diagnosis of ACS

Investigations:

ECG
A normal ECG does not exclude ACS
http://intermediate.frcemsuccess.com/rev/cardiology/ 2/5
7/31/2017 Cardiology - FRCEM Success
A normal ECG does not exclude ACS
Typical ECG ndings:
Regional ST-segment elevation (2 mm ST-elevation in two contiguous chest leads or 1 mm ST-
elevation in two contiguous limb leads)
New LBBB
Regional ST-segment depression (> 0.5 mm) or deep T wave inversion (> 2 mm deep)
Pathological Q waves
Serum troponin (I or T)
A detectable troponin level indicates damage to the myocardium
If initial troponin is raised (on admission or at 6 hours after symptom onset), repeat troponin
measurement 10 – 12 hours after onset of symptoms
Troponin is normally detectable within 12 hours following ACS and becomes undetectable 1 – 2 weeks
afterwards
Other conditions can also cause an increase in troponin e.g. arrhythmias, pericarditis, pulmonary
embolism, myocarditis
Consider a chest x-ray to exclude complications of ACS such as pulmonary oedema, or to exclude alternative
diagnoses such as pneumothorax or pneumonia

A diagnosis of myocardial infarction is de ned as the detection of the rise and/or fall of cardiac biomarkers
(preferably troponin) with at least 1 value above the 99th percentile of the upper reference limit, together with
evidence of myocardial ischaemia with at least one of the following:

Symptoms of ischaemia
ECG changes indicative of ischaemia (new ST-T changes or new LBBB)
Development of pathological Q waves in the ECG
Imaging evidence of new loss of viable myocardium or new regional wall motion abnormality

ECG lead changes:

A septal infarct shows changes in the V1 – V2 leads, and typically involves the left anterior descending
artery.
An anterior infarct shows changes in the V2 – V5 leads, and typically involves the left anterior descending
artery.
An anteroseptal infarct shows changes in the V1 – V4 leads and typically involves the left anterior
descending artery.
An anterolateral infarct shows changes in leads I, aVL, V3 – V6 (and reciprocal changes in leads II, III, aVF)
http://intermediate.frcemsuccess.com/rev/cardiology/ 3/5
7/31/2017 Cardiology - FRCEM Success
An anterolateral infarct shows changes in leads I, aVL, V3 – V6 (and reciprocal changes in leads II, III, aVF)
and typically involves the left anterior descending or the left circum ex artery.
An inferior infarct shows changes in leads II, III, aVF (and reciprocal changes in leads I, aVL) and typically
involves the right coronary artery.
A posterior infarct shows reciprocal changes in leads V1 – V3 and requires placement of posterior leads (V7
– V9) to identify ST-elevation – it typically involves the right coronary artery or the left circum ex artery.

Location of MI ECG leads Coronary vessel

Septal V1 – V2 LAD

Anterior V2 – V5 LAD

Anteroseptal V1 – V4 LAD

Anterolateral V3 – V6, I, aVL (R: II, III, aVF) LAD or left circum ex

Inferior II, III, aVF (R: I, aVL) RCA

Posterior R: V1 – V3 (P: V7 – V9) RCA or left circum ex

< Previous Next > Submit Something wrong?

FRCEM Success Resources


We are an online revision resource for The Royal College of Emergency Advanced Life Support Group
FRCEM Primary and Intermediate Medicine Emergency Medicine Journal
exam preparation. Irish Association for Emergency Lifeinthefastlane
Medicine Instant Anatomy
Terms & Conditions Advanced Trauma Life Support Patient.co.uk
Get in Touch Resuscitation Council (UK)
TeachMeAnatomy
http://intermediate.frcemsuccess.com/rev/cardiology/ 4/5
7/31/2017 Cardiology - FRCEM Success

Dashboard Subscription expires in: 82 Days Extend

Question Navigator
Cardiology
Question 13 of 24 1 Unanswered

A 45 year old man, known to inject drugs, and currently living in homeless hostels, presents 2 Unanswered
to AED complaining of malaise, fever, rigors and arthralgia. On auscultation he has a new
3 Unanswered
pansystolic murmur heard loudest over the left sternal edge.
4 Unanswered
a. What is the most likely causative infectious agent in this patient? (1 mark)
b. What is most likely valvular dysfunction in this patient and other than the murmur, give 5 Unanswered
two examination ndings of this type of valve disease? (1 mark)
6 Unanswered
c. Other than intravenous drug use, give two further risk factors for developing infective
endocarditis. (1 mark) 7 Unanswered

8 Unanswered

You did not answer this question 9 Unanswered

10 Unanswered

11 Unanswered
Answer
12 Unanswered
a. Staphylococcus aureus
b. Tricuspid regurgitation – Any two of:
Elevated JVP
Right ventricular heave
http://intermediate.frcemsuccess.com/rev/cardiology/ 1/4
7/31/2017 Cardiology - FRCEM Success

Hepatomegaly
Ascites
Peripheral oedema
c. Any two of:
Valvular heart disease
Prosthetic valves
Structural congenital heart disease
Rheumatic heart disease
Previous infective endocarditis
Hypertrophic cardiomyopathy
Invasive vascular procedures

Notes

Infective endocarditis:

Risk factors:

Valvular heart disease


Prosthetic valves
Structural congenital heart disease
Rheumatic heart disease
Previous infective endocarditis
Hypertrophic cardiomyopathy
Intravenous drug use (IVDU)
Invasive vascular procedures

Common organisms:

Staphylococcus aureus (most common cause overall – acute and subacute IE)
Streptococcus viridans (most common cause of subacute IE)
Pseudomonas aeruginosa
HACEK (Haemophilus spp.,Aggregatibacter actinomycetemcomitans, Cardiobacterium spp., Eikenella
corrodens, Kingella kingae)

Clinical features:
http://intermediate.frcemsuccess.com/rev/cardiology/ 2/4
7/31/2017 Cardiology - FRCEM Success
Clinical features:

Fever
New heart murmur
Non-speci c systemic features e.g. malaise, anorexia, weight loss, sweats, myalgia
Arthritis
Splenomegaly
Signs
Splinter (subungual) haemorrhages
Clubbing
Roth’s spots (retinal haemorrhages with pale centres)
Osler nodes (small tender red-to-purple nodules on the pulp of the terminal phalanges of the ngers
and toes)
Janeway’s lesions (irregular painless erythematous macules on the thenar and hypothenar eminence)

Investigations:

Echo – initially transthoracic, TOE is more sensitive


Blood cultures (x 3 at different sites and times, and preferably during fever)
FBC/CRP (nonspeci c raised in ammatory markers)

Management:

Empirical antibiotics according to local protocol


Refer urgently to cardiology

< Previous Next > Submit Something wrong?

FRCEM Success Resources


Advanced Life Support Group

http://intermediate.frcemsuccess.com/rev/cardiology/ 3/4
7/31/2017 Cardiology - FRCEM Success

Dashboard Subscription expires in: 82 Days Extend

Question Navigator
Cardiology
Question 14 of 24 1 Unanswered

A 61 year old woman, with a history of hypertension, presents to AED complaining of 2 Unanswered
sudden onset palpitations which came on about 2 hours ago. Her observations are: BP 125/95,
3 Unanswered
HR 150 bpm, RR 20, sats 98% OA, temp 36.7oC. Her ECG is shown below:
4 Unanswered

5 Unanswered

6 Unanswered

7 Unanswered

8 Unanswered

9 Unanswered

10 Unanswered

11 Unanswered

12 Unanswered

Courtesy of James Heilman, MD (Own work) [CC BY-SA 3.0 (http://creativecommons.org/licenses/by-


sa/3.0)], via Wikimedia Commons

http://intermediate.frcemsuccess.com/rev/cardiology/ 1/4
7/31/2017 Cardiology - FRCEM Success

a. What is the ECG diagnosis? (1 mark)


b. Give three drugs or classes of drugs which could be used for rate-control in this
condition. (1 mark)
c. You decide to chemically cardiovert the patient. Give two drugs you could use and
indicate which you would use if the patient had a history of structural heart disease. (1
mark)

You did not answer this question

Answer
a. Atrial brillation
b. Beta-blocker, calcium-channel blocker and digoxin
c. Flecainide and amiodarone (in structural heart disease)

Notes

Atrial brillation results from irregular, disorganised electrical activity in the atria, causing disorganised atrial
depolarisation and ineffective atrial contractions. The atrioventricular node receives more electrical impulses than
it can conduct, resulting in an irregular ventricular rhythm.

Most common causes:

Idiopathic
Ischaemic heart disease
Hypertension
Valvular heart disease
Hyperthyroidism

Clinical features:

http://intermediate.frcemsuccess.com/rev/cardiology/ 2/4
7/31/2017 Cardiology - FRCEM Success

Asymptomatic
Dyspnoea
Palpitations
Chest pain
Syncope/dizziness
Reduced exercise tolerance

Investigations:

ECG
Absent P-waves
Irregularly irregular ventricular rate (may be tachycardic)
Ambulatory 24-hour ECG
Bloods
FBC, CRP, U&Es, TFTs, LFTs, calcium, magnesium, glucose
Echo if suspicion of underlying structural heart disease
CXR if lung pathology is suspected

Management:

Identify and manage any underlying causes or triggers of AF


A rate-control treatment is recommended for most people with AF
Beta-blocker e.g. metoprolol (5 mg IV) or bisoprolol (5 mg PO)
Rate-limiting calcium-channel blocker e.g. diltiazem or verapamil (5 mg IV); if beta-blockers are
contraindicated or not tolerated
Digoxin 500 micrograms IV over 30 mins (only recommended for predominantly sedentary patients, for
patients in heart failure or in addition to other treatments where monotherapy is unsuccessful)
Rhythm control (electrical or pharmacological cardioversion) may be appropriate for people:
With new-onset AF (< 48 hours of onset)
Whose AF has a reversible cause (for example a chest infection)
Who have heart failure thought to be primarily caused, or worsened, by AF
With atrial utter and is considered suitable for an ablation strategy to restore sinus rhythm
For whom a rhythm control strategy would be more suitable based on clinical judgement
For cardioversion in patients with AF with onset < 48 hrs
Emergency electrical cardioversion in people with life-threatening haemodynamic instability caused by
new-onset AF
http://intermediate.frcemsuccess.com/rev/cardiology/ 3/4
7/31/2017 Cardiology - FRCEM Success
new-onset AF
Pharmacological
amiodarone (300 mg IV) in patients with evidence of structural heart disease,
ecainide (50 – 150 mg IV) in patients with no evidence of structural/ischaemic heart disease
If cardioversion is being considered in AF with onset > 48 hrs, it should be delayed until the person has been
maintained on therapeutic anticoagulation for a minimum of 3 weeks (and electrical cardioversion is
preferred)
Assess stroke risk using the CHA2DS2VASc assessment tool
Offer anticoagulation to all people with a CHA2DS2VASc score of 2 or above, and consider offering it
to men with a CHA2DS2VASc score of 1, after taking into account the person’s bleeding risk
Assess risk of bleeding using the HAS-BLED assessment tool
Anticoagulant options include warfarin and the new agents dabigatran, rivaroxaban or apixaban

< Previous Next > Submit Something wrong?

FRCEM Success Resources


We are an online revision resource for The Royal College of Emergency Advanced Life Support Group
FRCEM Primary and Intermediate Medicine Emergency Medicine Journal
exam preparation. Irish Association for Emergency Lifeinthefastlane
Medicine Instant Anatomy
Terms & Conditions Advanced Trauma Life Support Patient.co.uk
Get in Touch Resuscitation Council (UK)
TeachMeAnatomy
Trauma.org
Radiopaedia

©2014 - 2017 FRCEM Success | Website designed & hosted by Cyberfrog Design

http://intermediate.frcemsuccess.com/rev/cardiology/ 4/4
7/31/2017 Cardiology - FRCEM Success

Dashboard Subscription expires in: 82 Days Extend

Question Navigator
Cardiology
Question 15 of 24 1 Unanswered

A 65 year old man presents to AED with sudden onset ripping chest pain that radiates 2 Unanswered
through to his back. The pain is severe and was maximal at onset. His observations are: BP
3 Unanswered
160/90, HR 105 bpm, RR 20, sats 96% OA. You suspect aortic dissection.
4 Unanswered
a. What new murmur are you most likely to hear in this condition and give your rationale?
(1 mark) 5 Unanswered
b. Give three abnormal ndings you may see on a chest x-ray in this condition. (1 mark)
6 Unanswered
c. What is the most common risk factor for this condition? (1 mark)
7 Unanswered

8 Unanswered
You did not answer this question
9 Unanswered

10 Unanswered
Answer 11 Unanswered
a. Early diastolic murmur of aortic regurgitation
12 Unanswered
b. Any three of:
Widened/abnormal mediastinum
Double knuckle aorta
Left pleural effusion
http://intermediate.frcemsuccess.com/rev/cardiology/ 1/4
7/31/2017 Cardiology - FRCEM Success

Deviation of the trachea to the right


Separation of two parts of the wall of a calci ed aorta by > 5 mm
c. Hypertension

Notes

Aortic dissection is de ned as disruption of the medial layer of the aortic wall provoked by intramural bleeding,
resulting in separation of the aortic wall layers and subsequent formation of a true lumen and a false lumen, with or
without communication.

Risk factors:

Hypertension
Pre-existing aortic or aortic valve disease
Family history of aortic disease
Connective tissue disease (e.g. Marfan’s syndrome, Ehlers-Danlos syndrome)
Smoking
History of cardiac surgery
Direct blunt chest trauma

Clinical features:

Sudden tearing/ripping/sharp chest pain, maximal at onset, radiating to the back and interscapular area
Syncope
Hypertension (although hypotension may be present in tamponade)
Asymmetrical blood pressures in each arms
Inequality in pulses on each side
New diastolic murmur (aortic regurgitation)
Pericardial friction rub
Neurological de cit

Dissection may be classi ed as Stanford type A or B:

Type A involves the ascending aorta and/or aortic arch (35% mortality) – usually treated surgically
Type B involves only the descending aorta and arises distal to the origin of the left subclavian artery (15%
mortality) – usually treated medically
http://intermediate.frcemsuccess.com/rev/cardiology/ 2/4
7/31/2017 Cardiology - FRCEM Success
mortality) – usually treated medically

Investigations:

CXR
Widened/abnormal mediastinum
Double knuckle aorta
Left pleural effusion
Deviation of the trachea to the right
Separation of two parts of the wall of a calci ed aorta by > 5 mm
ECG – may show ischaemia as a consequence of aortic dissection
Echo
CT aorta – de nitive diagnosis

Complications:

Acute coronary ischaemia


Neurological de cit
Limb ischaemia
Cardiac tamponade
Aortic regurgitation
Haemothorax
Stroke
Aortic-oesophageal stula
Retroperitoneal haemorrhage
Mesenteric ischaemia
Renal failure

< Previous Next > Submit Something wrong?

FRCEM Success Resources


http://intermediate.frcemsuccess.com/rev/cardiology/ 3/4
7/31/2017 Cardiology - FRCEM Success

Dashboard Subscription expires in: 82 Days Extend

Question Navigator
Cardiology
Question 16 of 24 1 Unanswered

A 68 year old man is brought to hospital by his wife after he had a collapse this morning 2 Unanswered
with a transient loss of consciousness, whilst walking up a hill. On examination he has a loud
3 Unanswered
ejection systolic murmur heard loudest in the right second intercostal space
4 Unanswered
a. What is the most likely cause of his syncope? (1 mark)
b. Give the classical triad of this condition. (1 mark) 5 Unanswered
c. Give two further clinical features you might see on examination in this condition. (1
6 Unanswered
mark)
7 Unanswered

8 Unanswered
You did not answer this question
9 Unanswered

10 Unanswered
Answer 11 Unanswered
a. Aortic stenosis
12 Unanswered
b. Shortness of breath on exertion, angina and syncope
c. Any two of:
Slow-rising pulse
Narrow pulse pressure
http://intermediate.frcemsuccess.com/rev/cardiology/ 1/4
7/31/2017 Cardiology - FRCEM Success

Left ventricular heave


Palpable aortic thrill
Murmur radiates to carotids

Notes

Aortic stenosis:

Causes:

Advancing age
Congenital
Rheumatic heart disease

Clinical features:

Triad
SOBOE
Angina
Syncope
Slow-rising pulse
Narrow pulse pressure
Left ventricular heave
Palpable aortic thrill
Ejection systolic murmur heard loudest in the second right intercostal space, may radiate to the carotids

Investigations:

ECG
Left ventricular hypertrophy
Left ventricular strain (ST depression and T wave inversion I, avL, V5-V6)
Echo
Demonstrates presence and severity of valve disease

Syncope differential diagnosis:

http://intermediate.frcemsuccess.com/rev/cardiology/ 2/4
7/31/2017 Cardiology - FRCEM Success

Re ex syncope
Vasovagal syncope
Situational syncope e.g. cough, sneeze, micturition
Carotid sinus hypersensitivity (occurs when rotating the head)
Orthostatic hypotension
Hypovolaemia e.g. haemorrhage, dehydration, Addison’s disease
Cardiac arrhythmias
Sick sinus syndrome
Tachyarrhythmias
AV conduction disorders
Inherited syndromes e.g. long QT syndrome, Brugada syndrome
Structural cardiac disease
Hypertrophic obstructive cardiomyopathy
Obstructive cardiac valvular disease e.g. aortic stenosis
Acute aortic dissection
Pericardial disease or tamponade
Pulmonary embolism
Acute coronary syndrome
Substance abuse
Psychogenic

< Previous Next > Submit Something wrong?

FRCEM Success Resources


We are an online revision resource for The Royal College of Emergency Advanced Life Support Group
FRCEM Primary and Intermediate Medicine Emergency Medicine Journal
exam preparation. Irish Association for Emergency Lifeinthefastlane
Medicine Instant Anatomy
Terms & Conditions Advanced Trauma Life Support Patient.co.uk
Get in Touch Resuscitation Council (UK)
TeachMeAnatomy

http://intermediate.frcemsuccess.com/rev/cardiology/ 3/4
7/31/2017 Cardiology - FRCEM Success

Dashboard Subscription expires in: 82 Days Extend

Question Navigator
Cardiology
Question 17 of 24 1 Unanswered

A 72 year old man with a history of hypertension and angina is brought to AED with severe 2 Unanswered
central chest pain radiating down his left arm that has not been relieved with his usual GTN
3 Unanswered
spray, and has been ongoing now for about 1 hour. He has been given analgesia and an
antiemetic. His observations are: BP 148/85, HR 98, RR 17, sats 98% OA. His ECG con rms an 4 Unanswered
ST-elevation myocardial infarction.
5 Unanswered
a. Under what circumstances would brinolysis be indicated over PCI for treatment of
6 Unanswered
acute STEMI? (1 mark)
b. Give two absolute contraindications to brinolysis in ACS. (1 mark) 7 Unanswered
c. Following on from initial treatment, other than antiplatelets, give three further classes of
8 Unanswered
drugs that this patient will likely be discharged home with. (1 mark)
9 Unanswered

10 Unanswered
You did not answer this question
11 Unanswered

12 Unanswered
Answer
a. For patients presenting within 12 hours of onset of symptoms when primary PCI cannot be delivered within
120 minutes of the time when brinolysis could have been given
http://intermediate.frcemsuccess.com/rev/cardiology/ 1/4
7/31/2017 Cardiology - FRCEM Success

b. Any two of:


Prior intracranial haemorrhage
Known malignant intracranial lesion
Known structural cerebral vascular lesion e.g. arteriovenous malformation
Ischaemic stroke within previous 3 months
Suspected aortic dissection
Active bleeding or bleeding diathesis (excluding menses)
Signi cant closed head or facial trauma within previous 3 months
c. ACE inhibitor, beta-blocker (or calcium-channel blocker) and statin

Notes

Management of ACS:

If ACS is suspected:

Oxygen to maintain saturations of 94 – 98% (or 88 – 92% in patients at risk of hypercapnic respiratory
failure)
GTN (1 – 2 doses of sublingual 400 microgram spray or 300 microgram tablet)
Analgesia e.g. IV diamorphine 2.5 – 5 mg, given slowly over 5 minutes
Antiemetic e.g. IV metoclopramide 10 mg
Aspirin 300 mg (unless there is clear evidence of allergy)
If pain is ongoing, consider intravenous GTN (10 – 200 micrograms/min) or ISM (2 – 10 mg/hr) IV infusion

For diagnosed NSTEMI/Unstable angina (in addition to aspirin 300 mg):

Fondaparinux (2.5 mg SC) for patients without a high bleeding risk, unless coronary angiography is planned
within 24 hours
Unfractionated heparin is an alternative to fondaparinux if angiography is planned within 24 hours of
admission (or for patients with signi cant renal impairment)
For patients at low risk (mortality < 3%) offer 300 mg loading dose of clopidogrel only to those who may
undergo PCI within 24 h of admission to hospital
For patients at higher risk (mortality > 3%) offer 300 mg loading dose of clopidogrel to all patients; consider
adding a GPI (intravenous epti batide or tiro ban) if angiography is scheduled within 96 hours of hospital
admission
ADP receptor blocker alternatives to clopidogrel 300 mg include prasugrel 60 mg or ticagrelor 180 mg
http://intermediate.frcemsuccess.com/rev/cardiology/ 2/4
7/31/2017 Cardiology - FRCEM Success
ADP receptor blocker alternatives to clopidogrel 300 mg include prasugrel 60 mg or ticagrelor 180 mg
Additional drug therapy may include beta-blockers (diltiazem is alternative), ACE inhibitors and statins

For diagnosed STEMI (in addition to aspirin 300 mg):

Start antiplatelets/antithrombotics as per local protocol


For patients undergoing PPCI:
All patients undergoing PPCI should receive a platelet ADP receptor blocker e.g. clopidogrel 600
mg, ticagrelor 180 mg or prasugrel 60 mg
Anticoagulation with unfractionated or LMWH is given in the catheter lab, and in high risk cases a
glycoprotein IIb/IIIa inhibitor may also be given. Bivalirudin, a direct thrombin inhibitor may be
chosen as an alternative to heparin
For patients undergoing brinolysis:
Patients undergoing brinolysis should receive: clopidogrel 300 mg od and antithrombin therapy
(unfractionated heparin, LMWH or fondaparinux)
Offer coronary angiography, with follow-on primary PCI if indicated, as the preferred coronary reperfusion
strategy for people with acute STEMI if presentation is within 12 hours of onset of symptoms AND primary
PCI can be delivered within 120 minutes of the time when brinolysis could have been given
Consider coronary angiography, with follow-on primary PCI if indicated, for people with acute STEMI
presenting more than 12 hours after the onset of symptoms if there is evidence of continuing myocardial
ischaemia or of cardiogenic shock
For those who undergo brinolysis (indicated in those presenting within 12 hours of onset of symptoms in
whom primary PCI cannot be delivered in a reasonable timeframe), repeat an ECG 60 – 90 minutes after
administration. For those with residual ST-segment elevation, offer immediate coronary angiography, with
follow-on rescue PCI, if indicated

Relative contraindications to brinolysis:

History of chronic, severe, poorly controlled hypertension


Severe uncontrolled hypertension on presentation (SBP > 180 or DBP > 110)
History of prior ischemic stroke >3 months, dementia, or known intracranial pathology not covered in
contraindications
Traumatic or prolonged (>10 minutes) CPR or major surgery (< 3 weeks)
Recent (within 2 to 4 weeks) internal bleeding
Non-compressible vascular punctures
For streptokinase/anistreplase: prior exposure (>5 days ago) or prior allergic reaction to these agents
http://intermediate.frcemsuccess.com/rev/cardiology/ 3/4
7/31/2017 Cardiology - FRCEM Success
For streptokinase/anistreplase: prior exposure (>5 days ago) or prior allergic reaction to these agents
Pregnancy
Active peptic ulcer
Current use of anticoagulants: the higher the INR, the higher the risk of bleeding

< Previous Next > Submit Something wrong?

FRCEM Success Resources


We are an online revision resource for The Royal College of Emergency Advanced Life Support Group
FRCEM Primary and Intermediate Medicine Emergency Medicine Journal
exam preparation. Irish Association for Emergency Lifeinthefastlane
Medicine Instant Anatomy
Terms & Conditions Advanced Trauma Life Support Patient.co.uk
Get in Touch Resuscitation Council (UK)
TeachMeAnatomy
Trauma.org
Radiopaedia

©2014 - 2017 FRCEM Success | Website designed & hosted by Cyberfrog Design

http://intermediate.frcemsuccess.com/rev/cardiology/ 4/4
7/31/2017 Cardiology - FRCEM Success

Dashboard Subscription expires in: 82 Days Extend

Question Navigator
Cardiology
Question 18 of 24 1 Unanswered

A 65 year old man presents to AED with sudden onset ripping chest pain that radiates 2 Unanswered
through to his back. The pain is severe and was maximal at onset. His observations are: BP
3 Unanswered
208/110, HR 105 bpm, RR 20, sats 96% OA. You suspect aortic dissection.
4 Unanswered
a. Give two clinical features you might expect on examination of this patient that would
support your suspected diagnosis. (1 mark) 5 Unanswered
b. How can this condition be simply classi ed and de ne the classi cation? (1 mark)
6 Unanswered
c. Other than ‘death’ or ‘shock’, give two possible complications of this condition. (1 mark)
7 Unanswered

8 Unanswered
You did not answer this question
9 Unanswered

10 Unanswered
Answer 11 Unanswered
a. Any two of:
12 Unanswered
Asymmetrical upper limb blood pressures
Asymmetrical upper limb pulses
New diastolic murmur
Pericardial friction rub
http://intermediate.frcemsuccess.com/rev/cardiology/ 1/4
7/31/2017 Cardiology - FRCEM Success

b. Stanford classi cation: Type A involves the ascending aorta and/or aortic arch, Type B only involves the
descending aorta
c. Any two of:
Acute coronary ischaemia
Neurological de cit
Limb ischaemia
Cardiac tamponade
Aortic regurgitation
Haemothorax
Stroke
Aortic-oesophageal stula
Retroperitoneal haemorrhage
Mesenteric ischaemia
Renal failure

Notes

Aortic dissection is de ned as disruption of the medial layer of the aortic wall provoked by intramural bleeding,
resulting in separation of the aortic wall layers and subsequent formation of a true lumen and a false lumen, with or
without communication.

Risk factors:

Hypertension
Pre-existing aortic or aortic valve disease
Family history of aortic disease
Connective tissue disease (e.g. Marfan’s syndrome, Ehlers-Danlos syndrome)
Smoking
History of cardiac surgery
Direct blunt chest trauma

Clinical features:

Sudden tearing/ripping/sharp chest pain, maximal at onset, radiating to the back and interscapular area
Syncope
http://intermediate.frcemsuccess.com/rev/cardiology/ 2/4
7/31/2017 Cardiology - FRCEM Success

Hypertension (although hypotension may be present in tamponade)


Asymmetrical blood pressures in each arms
Inequality in pulses on each side
New diastolic murmur (aortic regurgitation)
Pericardial friction rub
Neurological de cit

Dissection may be classi ed as Stanford type A or B:

Type A involves the ascending aorta and/or aortic arch (35% mortality) – usually treated surgically
Type B involves only the descending aorta and arises distal to the origin of the left subclavian artery (15%
mortality) – usually treated medically

Investigations:

CXR
Widened/abnormal mediastinum
Double knuckle aorta
Left pleural effusion
Deviation of the trachea to the right
Separation of two parts of the wall of a calci ed aorta by > 5 mm
ECG – may show ischaemia as a consequence of aortic dissection
Echo
CT aorta – de nitive diagnosis

Complications:

Acute coronary ischaemia


Neurological de cit
Limb ischaemia
Cardiac tamponade
Aortic regurgitation
Haemothorax
Stroke
Aortic-oesophageal stula
Retroperitoneal haemorrhage
http://intermediate.frcemsuccess.com/rev/cardiology/ 3/4
7/31/2017 Cardiology - FRCEM Success
Retroperitoneal haemorrhage
Mesenteric ischaemia
Renal failure

< Previous Next > Submit Something wrong?

FRCEM Success Resources


We are an online revision resource for The Royal College of Emergency Advanced Life Support Group
FRCEM Primary and Intermediate Medicine Emergency Medicine Journal
exam preparation. Irish Association for Emergency Lifeinthefastlane
Medicine Instant Anatomy
Terms & Conditions Advanced Trauma Life Support Patient.co.uk
Get in Touch Resuscitation Council (UK)
TeachMeAnatomy
Trauma.org
Radiopaedia

©2014 - 2017 FRCEM Success | Website designed & hosted by Cyberfrog Design

http://intermediate.frcemsuccess.com/rev/cardiology/ 4/4
7/31/2017 Cardiology - FRCEM Success

Dashboard Subscription expires in: 82 Days Extend

Question Navigator
Cardiology
Question 19 of 24 1 Unanswered

A 62 year old overweight man presents to AED complaining of sudden onset left sided 2 Unanswered
chest pain ongoing now for about 20 minutes. He is nauseous and sweaty with the pain. He has a
3 Unanswered
history of hypertension and has smoked since he was a teenager. His observations are: BP 155/
90, HR 102, RR 16, sats 96% OA. His ECG is shown below: 4 Unanswered

5 Unanswered

6 Unanswered

7 Unanswered

8 Unanswered

9 Unanswered

10 Unanswered

11 Unanswered

12 Unanswered

http://intermediate.frcemsuccess.com/rev/cardiology/ 1/5
7/31/2017 Cardiology - FRCEM Success

Courtesy of James Heilman, MD (Own work) [CC BY-SA 4.0 (http://creativecommons.org/licenses/by-


sa/4.0)], via Wikimedia Commons

a. Describe the most obvious abnormalities on the ECG. (1 mark)


b. Give the ECG diagnosis and name the most likely anatomical structure affected in this
patient. (1 mark)
c. What is the most appropriate de nitive treatment for this patient? (1 mark)

You did not answer this question

Answer
a. ST-elevation in leads II, III and aVF AND reciprocal ST-depression in leads I and aVL
b. Inferior ST-elevation myocardial infarction (STEMI) – right coronary artery
c. Primary percutaneous coronary intervention (PCI)

Notes

In a patient with chest pain, acute coronary syndrome should be suspected if:

Pain in the chest or other areas (for example the arms, back, or jaw) lasts longer than 15 minutes
Chest pain is associated with nausea and vomiting, sweating or breathlessness, or a combination of these
Chest pain is associated with haemodynamic instability (for example the person has a systolic blood
pressure less than 90 mmHg)
Chest pain is of new-onset, or is the result of an abrupt deterioration of stable angina; with pain occurring
frequently with little or no exertion, and often lasting longer than 15 minutes
The patient’s response to GTN should not be used to con rm or exclude a diagnosis of ACS

Investigations:

ECG
A normal ECG does not exclude ACS
http://intermediate.frcemsuccess.com/rev/cardiology/ 2/5
7/31/2017 Cardiology - FRCEM Success
A normal ECG does not exclude ACS
Typical ECG ndings:
Regional ST-segment elevation (2 mm ST-elevation in two contiguous chest leads or 1 mm ST-
elevation in two contiguous limb leads)
New LBBB
Regional ST-segment depression (> 0.5 mm) or deep T wave inversion (> 2 mm deep)
Pathological Q waves
Serum troponin (I or T)
A detectable troponin level indicates damage to the myocardium
If initial troponin is raised (on admission or at 6 hours after symptom onset), repeat troponin
measurement 10 – 12 hours after onset of symptoms
Troponin is normally detectable within 12 hours following ACS and becomes undetectable 1 – 2 weeks
afterwards
Other conditions can also cause an increase in troponin e.g. arrhythmias, pericarditis, pulmonary
embolism, myocarditis
Consider a chest x-ray to exclude complications of ACS such as pulmonary oedema, or to exclude alternative
diagnoses such as pneumothorax or pneumonia

A diagnosis of myocardial infarction is de ned as the detection of the rise and/or fall of cardiac biomarkers
(preferably troponin) with at least 1 value above the 99th percentile of the upper reference limit, together with
evidence of myocardial ischaemia with at least one of the following:

Symptoms of ischaemia
ECG changes indicative of ischaemia (new ST-T changes or new LBBB)
Development of pathological Q waves in the ECG
Imaging evidence of new loss of viable myocardium or new regional wall motion abnormality

ECG lead changes:

A septal infarct shows changes in the V1 – V2 leads, and typically involves the left anterior descending
artery.
An anterior infarct shows changes in the V2 – V5 leads, and typically involves the left anterior descending
artery.
An anteroseptal infarct shows changes in the V1 – V4 leads and typically involves the left anterior
descending artery.
An anterolateral infarct shows changes in leads I, aVL, V3 – V6 (and reciprocal changes in leads II, III, aVF)
http://intermediate.frcemsuccess.com/rev/cardiology/ 3/5
7/31/2017 Cardiology - FRCEM Success
An anterolateral infarct shows changes in leads I, aVL, V3 – V6 (and reciprocal changes in leads II, III, aVF)
and typically involves the left anterior descending or the left circum ex artery.
An inferior infarct shows changes in leads II, III, aVF (and reciprocal changes in leads I, aVL) and typically
involves the right coronary artery.
A posterior infarct shows reciprocal changes in leads V1 – V3 and requires placement of posterior leads (V7
– V9) to identify ST-elevation – it typically involves the right coronary artery or the left circum ex artery.

Location of MI ECG leads Coronary vessel

Septal V1 – V2 LAD

Anterior V2 – V5 LAD

Anteroseptal V1 – V4 LAD

Anterolateral V3 – V6, I, aVL (R: II, III, aVF) LAD or left circum ex

Inferior II, III, aVF (R: I, aVL) RCA

Posterior R: V1 – V3 (P: V7 – V9) RCA or left circum ex

< Previous Next > Submit Something wrong?

FRCEM Success Resources


We are an online revision resource for The Royal College of Emergency Advanced Life Support Group
FRCEM Primary and Intermediate Medicine Emergency Medicine Journal
exam preparation. Irish Association for Emergency Lifeinthefastlane
Medicine Instant Anatomy
Terms & Conditions Advanced Trauma Life Support Patient.co.uk
Get in Touch Resuscitation Council (UK)
TeachMeAnatomy

http://intermediate.frcemsuccess.com/rev/cardiology/ 4/5
7/31/2017 Cardiology - FRCEM Success

Dashboard Subscription expires in: 82 Days Extend

Question Navigator
Cardiology
Question 20 of 24 1 Unanswered

A 39 year old woman presents to AED complaining of a sharp retrosternal chest pain which 2 Unanswered
has been ongoing for about 48 hours. The pain feels worse when she lies down at night, and
3 Unanswered
when she coughs or breathes deeply.
4 Unanswered
a. Other than infection, give two possible causes of acute pericarditis. (1 mark)
b. What classical triad of clinical features would indicate cardiac tamponade? (1 mark) 5 Unanswered
c. Give two investigations that could be performed in AED that would allow you to identify
6 Unanswered
pericardial effusion. (1 mark)
7 Unanswered

8 Unanswered
You did not answer this question
9 Unanswered

10 Unanswered
Answer 11 Unanswered
a. Any two of:
12 Unanswered
Myocardial infarction
Dressler’s syndrome
Malignancy
Rheumatic fever
http://intermediate.frcemsuccess.com/rev/cardiology/ 1/4
7/31/2017 Cardiology - FRCEM Success

Uraemia
Collagen vascular disease e.g. rheumatoid arthritis
Uraemia
Post-surgery, trauma or radiotherapy
Drugs (e.g. hydralazine, isoniazid, phenytoin, anticoagulants)
b. Beck’s triad: Hypotension, muf ed heart sounds and jugular venous distension
c. Chest x-ray and bedside echocardiogram

Notes

Acute pericarditis:

Causes:

Myocardial infarction (including Dressler’s syndrome)


Viral infection (e.g. Coxsackie, mumps, EBV, CMV, HIV)
Bacterial infection (e.g. Staphylococcus, Haemophilus)
Tuberculosis
Malignancy (e.g. bronchus or breast locally invading)
Rheumatic fever
Uraemia
Collagen vascular disease (e.g. SLE, RA)
Post-surgery, radiotherapy or trauma
Drugs (e.g. hydralazine, isoniazid, phenytoin, anticoagulants)

Clinical features:

Sharp, constant sternal pain relieved by sitting forwards, pain may radiate to the left shoulder and arm, and
is worse when lying on the left side and on inspiration, swallowing and coughing
Fever, cough, arthralgia
In cardiac tamponade – breathlessness, dysphagia, cough and hoarseness
Pericardial friction rub (high-pitched scratching sound, best heard over the left sternal border during
expiration)
In cardiac tamponade – pulsus paradoxus (decease in palpable pulse and arterial systolic BP of 10 mmHg on
inspiration), hypotension, muf ed heart sounds and jugular venous distension (Beck’s triad)

http://intermediate.frcemsuccess.com/rev/cardiology/ 2/4
7/31/2017 Cardiology - FRCEM Success

Investigations:

ECG
Sinus tachycardia
Widespread, saddle-shaped ST-elevation (in at least two limb leads and all chest leads)
ST-depression in leads aVR or I
PR depression
Tall, peaked T waves
QRS has decreased amplitude in pericardial effusion
CXR
Usually normal but may show a globular heart due to effusion, or associated pathology
Bloods
Troponin, FBC, CRP, U&E
Echo
May demonstrate a pericardial effusion

Management is dependent on the cause to an extent but usually involves treatment with NSAIDs.

< Previous Next > Submit Something wrong?

FRCEM Success Resources


We are an online revision resource for The Royal College of Emergency Advanced Life Support Group
FRCEM Primary and Intermediate Medicine Emergency Medicine Journal
exam preparation. Irish Association for Emergency Lifeinthefastlane
Medicine Instant Anatomy
Terms & Conditions Advanced Trauma Life Support Patient.co.uk
Get in Touch Resuscitation Council (UK)
TeachMeAnatomy
Trauma.org
Radiopaedia

http://intermediate.frcemsuccess.com/rev/cardiology/ 3/4
7/31/2017 Cardiology - FRCEM Success

Dashboard Subscription expires in: 82 Days Extend

Question Navigator
Cardiology
Question 21 of 24 1 Unanswered

A 72 year old female patient, with a past medical history only of hypertension, presented 2 Unanswered
with a fever and productive cough and has been diagnosed with community acquired pneumonia.
3 Unanswered
A nurse asks you to review a routine ECG which is shown below:
4 Unanswered

5 Unanswered

6 Unanswered

7 Unanswered

8 Unanswered

9 Unanswered

10 Unanswered

11 Unanswered

12 Unanswered

Courtesy of CardioNetworks: [ ] (CardioNetworks: A b_ecg.jpg) [CC BY-SA 3.0


(http://creativecommons.org/licenses/by-sa/3.0)], via Wikimedia Commons

http://intermediate.frcemsuccess.com/rev/cardiology/ 1/5
7/31/2017 Cardiology - FRCEM Success

a. Describe the two most obvious abnormalities on this ECG and give the diagnosis. (1
mark)
b. What is the most concerning complication of this condition? (1 mark)
c. You are considering thromboprophylaxis in this patient. Calculate this patient’s
CHA2DS2VASc score. (1 mark)

You did not answer this question

Answer
a. Irregularly irregular rhythm and absent P waves – atrial brillation
b. Increased risk of stroke/TIA
c. Hypertension (1 point) + Age 65 – 74 years (1 point) + Female (1 point) = 3 points

Notes

Atrial brillation results from irregular, disorganised electrical activity in the atria, causing disorganised atrial
depolarisation and ineffective atrial contractions. The atrioventricular node receives more electrical impulses than
it can conduct, resulting in an irregular ventricular rhythm.

Most common causes:

Idiopathic
Ischaemic heart disease
Hypertension
Valvular heart disease
Hyperthyroidism

Clinical features:

Asymptomatic
http://intermediate.frcemsuccess.com/rev/cardiology/ 2/5
7/31/2017 Cardiology - FRCEM Success

Dyspnoea
Palpitations
Chest pain
Syncope/dizziness
Reduced exercise tolerance

Investigations:

ECG
Absent P-waves
Irregularly irregular ventricular rate (may be tachycardic)
Ambulatory 24-hour ECG
Bloods
FBC, CRP, U&Es, TFTs, LFTs, calcium, magnesium, glucose
Echo if suspicion of underlying structural heart disease
CXR if lung pathology is suspected

Management:

Identify and manage any underlying causes or triggers of AF


A rate-control treatment is recommended for most people with AF
Beta-blocker e.g. metoprolol (5 mg IV) or bisoprolol (5 mg PO)
Rate-limiting calcium-channel blocker e.g. diltiazem or verapamil (5 mg IV); if beta-blockers are
contraindicated or not tolerated
Digoxin 500 micrograms IV over 30 mins (only recommended for predominantly sedentary patients, for
patients in heart failure or in addition to other treatments where monotherapy is unsuccessful)
Rhythm control (electrical or pharmacological cardioversion) may be appropriate for people:
With new-onset AF (< 48 hours of onset)
Whose AF has a reversible cause (for example a chest infection)
Who have heart failure thought to be primarily caused, or worsened, by AF
With atrial utter and is considered suitable for an ablation strategy to restore sinus rhythm
For whom a rhythm control strategy would be more suitable based on clinical judgement
For cardioversion in patients with AF with onset < 48 hrs
Emergency electrical cardioversion in people with life-threatening haemodynamic instability caused by
new-onset AF
Pharmacological
http://intermediate.frcemsuccess.com/rev/cardiology/ 3/5
7/31/2017 Cardiology - FRCEM Success
Pharmacological
amiodarone (300 mg IV) in patients with evidence of structural heart disease,
ecainide (50 – 150 mg IV) in patients with no evidence of structural/ischaemic heart disease
If cardioversion is being considered in AF with onset > 48 hrs, it should be delayed until the person has been
maintained on therapeutic anticoagulation for a minimum of 3 weeks (and electrical cardioversion is
preferred)
Assess stroke risk using the CHA2DS2VASc assessment tool
Offer anticoagulation to all people with a CHA2DS2VASc score of 2 or above, and consider offering it
to men with a CHA2DS2VASc score of 1, after taking into account the person’s bleeding risk
Assess risk of bleeding using the HAS-BLED assessment tool
Anticoagulant options include warfarin and the new agents dabigatran, rivaroxaban or apixaban

CHA2DS2VASc score tool:

Congestive heart failure/left ventricular dysfunction (1 point)


Hypertension (1 point)
Age >= 75 years (2 points)
Diabetes mellitus (1 point)
Stroke/TIA/Thromboembolism (2 points)
Vascular disease (1 point)
Age 65 – 74 years (1 point)
Sex category female (1 point)

HAS-BLED tool (A score of 1 point should be given for each of the following risk factors):

Hypertension (uncontrolled, greater than 160 mmHg systolic)


Abnormal liver function
Abnormal renal function
Stroke (previous history, particularly lacunar)
Bleeding (bleeding history or predisposition)
Labile international normalized ratios (INRs, therapeutic time in range less than 60%)
Elderly (aged over 65 years)
Drugs (antiplatelet agents or nonsteroidal anti-in ammatory drugs)
Harmful alcohol consumption

http://intermediate.frcemsuccess.com/rev/cardiology/ 4/5
7/31/2017 Cardiology - FRCEM Success

Dashboard Subscription expires in: 82 Days Extend

Question Navigator
Cardiology
Question 22 of 24 1 Unanswered

A 71 year old man, with a history of hypertension and diabetes, presents to AED 2 Unanswered
complaining of central crushing chest pain that has been ongoing for about 25 minutes. His
3 Unanswered
observations are: BP 145/80, HR 105, RR 18, sats 96 % OA. He has been given aspirin 300 mg by
triage. 4 Unanswered

a. In which ECG leads is T wave inversion a normal variant? (1 mark) 5 Unanswered


b. What nding on ECG is speci cally suggestive of current OR old myocardial infarction?
6 Unanswered
(1 mark)
c. An ECG con rms Non ST-elevation ischaemic change (NSTEMI). Give two further drugs 7 Unanswered
(including route and dose) this patient should be given in AED. (1 mark)
8 Unanswered

9 Unanswered
You did not answer this question
10 Unanswered

11 Unanswered

Answer 12 Unanswered

a. Leads III, aVR and V1


b. Pathological Q waves
c. Clopidogrel 300 mg (or prasugrel 60 mg or ticagrelor 180 mg) orally and fondaparinux 2.5 mg
http://intermediate.frcemsuccess.com/rev/cardiology/ 1/3
7/31/2017 Cardiology - FRCEM Success

subcutaneously (N.B. unfractionated heparin is an alternative)

Notes

Management of ACS:

If ACS is suspected:

Oxygen to maintain saturations of 94 – 98% (or 88 – 92% in patients at risk of hypercapnic respiratory
failure)
GTN (1 – 2 doses of sublingual 400 microgram spray or 300 microgram tablet)
Analgesia e.g. IV diamorphine 2.5 – 5 mg, given slowly over 5 minutes
Antiemetic e.g. IV metoclopramide 10 mg
Aspirin 300 mg (unless there is clear evidence of allergy)
If pain is ongoing, consider intravenous GTN (10 – 200 micrograms/min) or ISM (2 – 10 mg/hr) IV infusion

For diagnosed NSTEMI/Unstable angina (in addition to aspirin 300 mg):

Fondaparinux (2.5 mg SC) for patients without a high bleeding risk, unless coronary angiography is planned
within 24 hours
Unfractionated heparin is an alternative to fondaparinux if angiography is planned within 24 hours of
admission (or for patients with signi cant renal impairment)
For patients at low risk (mortality < 3%) offer 300 mg loading dose of clopidogrel only to those who may
undergo PCI within 24 h of admission to hospital
For patients at higher risk (mortality > 3%) offer 300 mg loading dose of clopidogrel to all patients; consider
adding a GPI (intravenous epti batide or tiro ban) if angiography is scheduled within 96 hours of hospital
admission
ADP receptor blocker alternatives to clopidogrel 300 mg include prasugrel 60 mg or ticagrelor 180 mg
Additional drug therapy may include beta-blockers (diltiazem is alternative), ACE inhibitors and statins

For diagnosed STEMI (in addition to aspirin 300 mg):

Start antiplatelets/antithrombotics as per local protocol


For patients undergoing PPCI:
All patients undergoing PPCI should receive a platelet ADP receptor blocker e.g. clopidogrel 600
mg, ticagrelor 180 mg or prasugrel 60 mg
http://intermediate.frcemsuccess.com/rev/cardiology/ 2/3
7/31/2017 Cardiology - FRCEM Success
mg, ticagrelor 180 mg or prasugrel 60 mg
Anticoagulation with unfractionated or LMWH is given in the catheter lab, and in high risk cases a
glycoprotein IIb/IIIa inhibitor may also be given. Bivalirudin, a direct thrombin inhibitor may be
chosen as an alternative to heparin
For patients undergoing brinolysis:
Patients undergoing brinolysis should receive: clopidogrel 300 mg od and antithrombin therapy
(unfractionated heparin, LMWH or fondaparinux)
Offer coronary angiography, with follow-on primary PCI if indicated, as the preferred coronary reperfusion
strategy for people with acute STEMI if presentation is within 12 hours of onset of symptoms AND primary
PCI can be delivered within 120 minutes of the time when brinolysis could have been given
Consider coronary angiography, with follow-on primary PCI if indicated, for people with acute STEMI
presenting more than 12 hours after the onset of symptoms if there is evidence of continuing myocardial
ischaemia or of cardiogenic shock
For those who undergo brinolysis (indicated in those presenting within 12 hours of onset of symptoms in
whom primary PCI cannot be delivered in a reasonable timeframe), repeat an ECG 60 – 90 minutes after
administration. For those with residual ST-segment elevation, offer immediate coronary angiography, with
follow-on rescue PCI, if indicated

< Previous Next > Submit Something wrong?

FRCEM Success Resources


We are an online revision resource for The Royal College of Emergency Advanced Life Support Group
FRCEM Primary and Intermediate Medicine Emergency Medicine Journal
exam preparation. Irish Association for Emergency Lifeinthefastlane
Medicine Instant Anatomy
Terms & Conditions Advanced Trauma Life Support Patient.co.uk
Get in Touch Resuscitation Council (UK)
TeachMeAnatomy
Trauma.org
Radiopaedia

©2014 - 2017 FRCEM Success | Website designed & hosted by Cyberfrog Design

http://intermediate.frcemsuccess.com/rev/cardiology/ 3/3
7/31/2017 Cardiology - FRCEM Success

Dashboard Subscription expires in: 82 Days Extend

Question Navigator
Cardiology
Question 23 of 24 1 Unanswered

A 66 year old overweight man presents to AED complaining of sudden onset left sided 2 Unanswered
chest pain ongoing now for about 20 minutes. He is nauseous and sweaty with the pain. He has a
3 Unanswered
history of hypertension and has smoked since he was a teenager. His observations are: BP 155/
90, HR 102, RR 16, sats 96% OA. On admission he has had a chest x-ray, blood tests including a 4 Unanswered
serum troponin and an ECG which are all normal.
5 Unanswered
a. Other than ST-elevation, what nding on ECG could indicate a STEMI? (1 mark)
6 Unanswered
b. Initial serum troponin and ECG performed on admission are normal. How would you
further investigate this patient? (1 mark) 7 Unanswered
c. You are asked to calculate his TIMI score. Give two criteria that would score a point in
8 Unanswered
the TIMI scoring system. (1 mark)
9 Unanswered

10 Unanswered
You did not answer this question
11 Unanswered

12 Unanswered
Answer
a. New left bundle branch block (LBBB)
b. Repeat serum troponin at 6 hours after symptom onset and serial ECGs to detect ischaemic change
http://intermediate.frcemsuccess.com/rev/cardiology/ 1/3
7/31/2017 Cardiology - FRCEM Success

c. Any two of: (N.B. TIMI risk score for Unstable angina/NSTEMI helps stratify patients with anginal
symptoms)
Age > 65 years
Risk factor for coronary artery disease (FHx, HTN, hyperlipidaemia, DM, smoker)
Known coronary artery disease with stenosis >= 50%
Aspirin use in last 7 days
Recent episode of angina prior to this event
Raised troponin level
ST segment deviation >= 0.5 mm on ECG

Notes

In a patient with chest pain, acute coronary syndrome should be suspected if:

Pain in the chest or other areas (for example the arms, back, or jaw) lasts longer than 15 minutes
Chest pain is associated with nausea and vomiting, sweating or breathlessness, or a combination of these
Chest pain is associated with haemodynamic instability (for example the person has a systolic blood
pressure less than 90 mmHg)
Chest pain is of a new-onset, or is the result of an abrupt deterioration of stable angina; with pain occurring
frequently with little or no exertion, and often lasting longer than 15 minutes
The patient’s response to GTN should not be used to con rm or exclude a diagnosis of ACS

Investigations:

ECG
A normal ECG does not exclude ACS
Typical ECG ndings:
Regional ST-segment elevation (2 mm ST-elevation in two contiguous chest leads or 1 mm ST-
elevation in two contiguous limb leads)
New LBBB
Regional ST-segment depression (> 0.5 mm) or deep T wave inversion (> 2 mm deep)
Pathological Q waves
Serum troponin (I or T)
A detectable troponin level indicates damage to the myocardium
If initial troponin is raised (on admission, or at 6 hours after symptom onset), repeat troponin
measurement 10 – 12 hours after onset of symptoms
http://intermediate.frcemsuccess.com/rev/cardiology/ 2/3
7/31/2017 Cardiology - FRCEM Success
measurement 10 – 12 hours after onset of symptoms
Troponin is normally detectable within 12 hours following ACS and becomes undetectable 1 – 2 weeks
afterwards
Other conditions can also cause an increase in troponin e.g. arrhythmias, pericarditis, pulmonary
embolism, myocarditis
Consider a chest x-ray to exclude complications of ACS such as pulmonary oedema, or to exclude alternative
diagnoses such as pneumothorax or pneumonia

A diagnosis of myocardial infarction is de ned as the detection of the rise and/or fall of cardiac biomarkers
(preferably troponin) with at least 1 value above the 99th percentile of the upper reference limit, together with
evidence of myocardial ischaemia with at least one of the following:

Symptoms of ischaemia
ECG changes indicative of ischaemia (new ST-T changes or new LBBB)
Development of pathological Q waves in the ECG
Imaging evidence of new loss of viable myocardium or new regional wall motion abnormality

< Previous Next > Submit Something wrong?

FRCEM Success Resources


We are an online revision resource for The Royal College of Emergency Advanced Life Support Group
FRCEM Primary and Intermediate Medicine Emergency Medicine Journal
exam preparation. Irish Association for Emergency Lifeinthefastlane
Medicine Instant Anatomy
Terms & Conditions Advanced Trauma Life Support Patient.co.uk
Get in Touch Resuscitation Council (UK)
TeachMeAnatomy
Trauma.org
Radiopaedia

©2014 - 2017 FRCEM Success | Website designed & hosted by Cyberfrog Design

http://intermediate.frcemsuccess.com/rev/cardiology/ 3/3
7/31/2017 Cardiology - FRCEM Success

Dashboard Subscription expires in: 82 Days Extend

Question Navigator
Cardiology
Question 24 of 24 1 Unanswered

A 72 year old female patient, with a past medical history only of hypertension, presented 2 Unanswered
with a fever and productive cough. Blood tests demonstrate a raised WCC and CRP, and chest x-
3 Unanswered
ray demonstrates consolidation. A nurse asks you to review a routine ECG which is shown below:
4 Unanswered

5 Unanswered

6 Unanswered

7 Unanswered

8 Unanswered

9 Unanswered

10 Unanswered

11 Unanswered

12 Unanswered

Courtesy of CardioNetworks: [ ] (CardioNetworks: A b_ecg.jpg) [CC BY-SA 3.0


(http://creativecommons.org/licenses/by-sa/3.0)], via Wikimedia Commons

http://intermediate.frcemsuccess.com/rev/cardiology/ 1/6
7/31/2017 Cardiology - FRCEM Success

a. A review of the patient’s notes shows these changes are new. Give four further blood
tests that you would request in this patient? (1 mark)
b. You are considering thromboprophylaxis in this patient. How could you calculate the
patient’s risk of bleeding, and give two risk factors that would score a point? (1 mark)
c. It is decided the patient should be started on apixaban. What is the mode of action of
apixaban? (1 mark)

You did not answer this question

Answer
a. Any four of:
U&Es
LFTs
TFTs
Ca2+
Mg2+
Glucose
b. HAS-BLED score, any two of the following:
Hypertension (uncontrolled, greater than 160 mmHg systolic)
Abnormal liver function
Abnormal renal function
Stroke (previous history, particularly lacunar)
Bleeding (bleeding history or predisposition)
Labile international normalized ratios (INRs, therapeutic time in range less than 60%)
Elderly (aged over 65 years)
Drugs (antiplatelet agents or nonsteroidal anti-in ammatory drugs)
Harmful alcohol consumption
c. Apixaban is a direct inhibitor of factor Xa (N.B. Apixaban and rivaroxaban are direct inhibitors of factor X,
dabigratan is a reversible inhibitor of free thrombin, brin-bound thrombin and thrombin-induced platelet
http://intermediate.frcemsuccess.com/rev/cardiology/ 2/6
7/31/2017 Cardiology - FRCEM Success
dabigratan is a reversible inhibitor of free thrombin, brin-bound thrombin and thrombin-induced platelet
aggregation)

Notes

Atrial brillation results from irregular, disorganised electrical activity in the atria, causing disorganised atrial
depolarisation and ineffective atrial contractions. The atrioventricular node receives more electrical impulses than
it can conduct, resulting in an irregular ventricular rhythm.

Most common causes:

Idiopathic
Ischaemic heart disease
Hypertension
Valvular heart disease
Hyperthyroidism

Clinical features:

Asymptomatic
Dyspnoea
Palpitations
Chest pain
Syncope/dizziness
Reduced exercise tolerance

Investigations:

ECG
Absent P-waves
Irregularly irregular ventricular rate (may be tachycardic)
Ambulatory 24-hour ECG
Bloods
FBC, CRP, U&Es, TFTs, LFTs, calcium, magnesium, glucose
Echo if suspicion of underlying structural heart disease
CXR if lung pathology is suspected

http://intermediate.frcemsuccess.com/rev/cardiology/ 3/6
7/31/2017 Cardiology - FRCEM Success

Management:

Identify and manage any underlying causes or triggers of AF


A rate-control treatment is recommended for most people with AF
Beta-blocker e.g. metoprolol (5 mg IV) or bisoprolol (5 mg PO)
Rate-limiting calcium-channel blocker e.g. diltiazem or verapamil (5 mg IV); if beta-blockers are
contraindicated or not tolerated
Digoxin 500 micrograms IV over 30 mins (only recommended for predominantly sedentary patients, for
patients in heart failure or in addition to other treatments where monotherapy is unsuccessful)
Rhythm control (electrical or pharmacological cardioversion) may be appropriate for people:
With new-onset AF (< 48 hours of onset)
Whose AF has a reversible cause (for example a chest infection)
Who have heart failure thought to be primarily caused, or worsened, by AF
With atrial utter and is considered suitable for an ablation strategy to restore sinus rhythm
For whom a rhythm control strategy would be more suitable based on clinical judgement
For cardioversion in patients with AF with onset < 48 hrs
Emergency electrical cardioversion in people with life-threatening haemodynamic instability caused by
new-onset AF
Pharmacological
amiodarone (300 mg IV) in patients with evidence of structural heart disease,
ecainide (50 – 150 mg IV) in patients with no evidence of structural/ischaemic heart disease
If cardioversion is being considered in AF with onset > 48 hrs, it should be delayed until the person has been
maintained on therapeutic anticoagulation for a minimum of 3 weeks (and electrical cardioversion is
preferred)
Assess stroke risk using the CHA2DS2VASc assessment tool
Offer anticoagulation to all people with a CHA2DS2VASc score of 2 or above, and consider offering it
to men with a CHA2DS2VASc score of 1, after taking into account the person’s bleeding risk
Assess risk of bleeding using the HAS-BLED assessment tool
Anticoagulant options include warfarin and the new agents dabigatran, rivaroxaban or apixaban

CHA2DS2VASc score tool:

Congestive heart failure/left ventricular dysfunction (1 point)


Hypertension (1 point)
Age >= 75 years (2 points)
http://intermediate.frcemsuccess.com/rev/cardiology/ 4/6
7/31/2017 Cardiology - FRCEM Success
Age >= 75 years (2 points)
Diabetes mellitus (1 point)
Stroke/TIA/Thromboembolism (2 points)
Vascular disease (1 point)
Age 65 – 74 years (1 point)
Sex category female (1 point)

HAS-BLED tool (A score of 1 point should be given for each of the following risk factors):

Hypertension (uncontrolled, greater than 160 mmHg systolic)


Abnormal liver function
Abnormal renal function
Stroke (previous history, particularly lacunar)
Bleeding (bleeding history or predisposition)
Labile international normalized ratios (INRs, therapeutic time in range less than 60%)
Elderly (aged over 65 years)
Drugs (antiplatelet agents or nonsteroidal anti-in ammatory drugs)
Harmful alcohol consumption

< Previous Submit Something wrong?

FRCEM Success Resources


We are an online revision resource for The Royal College of Emergency Advanced Life Support Group
FRCEM Primary and Intermediate Medicine Emergency Medicine Journal
exam preparation. Irish Association for Emergency Lifeinthefastlane
Medicine Instant Anatomy
Terms & Conditions Advanced Trauma Life Support Patient.co.uk
Get in Touch Resuscitation Council (UK)
TeachMeAnatomy
Trauma.org
Radiopaedia

http://intermediate.frcemsuccess.com/rev/cardiology/ 5/6

You might also like